PDA

View Full Version : Science For Science! - Learning new things about Physics, Chemistry, Statistics, and so on



Pages : [1] 2 3 4 5 6

Yora
2014-02-25, 09:45 AM
We had some threads like this in the past and they were really quite fun and educational. So let's do it again.

First question is going right into the deep end: Quantum tunneling is the effect of subatomic particles popping in and out of existance within a very small region of space, which can cause them to end up on the other side of a barrier, without physically moving through it.
Can electrons quantum tunnel out of a wire and fly away, being effectively removed from the electonic current, resulting in a (localized) loss of energy?
Or is the amount of electrons in a wire just so big and the outermost layer where electrons could be lost so thin, that the effect is pretty much undetectable?

shawnhcorey
2014-02-25, 10:07 AM
Quantum Tunnelling effects all electrons regardless of their position in the wire. And there are so many electrons that if one tunnelled away, it would be quickly replaced.

Jay R
2014-02-25, 10:39 AM
We had some threads like this in the past and they were really quite fun and educational. So let's do it again.

First question is going right into the deep end: Quantum tunneling is the effect of subatomic particles popping in and out of existance within a very small region of space, which can cause them to end up on the other side of a barrier, without physically moving through it.
Can electrons quantum tunnel out of a wire and fly away, being effectively removed from the electonic current, resulting in a (localized) loss of energy?
Or is the amount of electrons in a wire just so big and the outermost layer where electrons could be lost so thin, that the effect is pretty much undetectable?

The biggest difficulty in understanding quantum effects is that it is almost impossible for us to really comprehend the scale.

Yes, it's theoretically possible, but the distance to travel in tunneling through the wire's insulating layer is incredibly far. Most tunneling occurs on the range no more than 1-3 nanometers. It's based on the probability density function of the electron, which has incredibly low values at distances needed to tunnel through even the thinnest insulating sheath.

Simple answer: Quantum effects are usually subatomic. Don't try to apply quantum mechanics on a macro level, or you'll eventually wind up wondering why a living cat cannot observe itself inside the box.

shawnhcorey
2014-02-25, 11:00 AM
Simple answer: Quantum effects are usually subatomic. Don't try to apply quantum mechanics on a macro level, or you'll eventually wind up wondering why a living cat cannot observe itself inside the box.

That's a bad attitude. Chemistry works because of quantum effects. Electronics work because of quantum effects. Quantum effects everyday life.

the_druid_droid
2014-02-25, 12:17 PM
We had some threads like this in the past and they were really quite fun and educational. So let's do it again.

First question is going right into the deep end: Quantum tunneling is the effect of subatomic particles popping in and out of existance within a very small region of space, which can cause them to end up on the other side of a barrier, without physically moving through it.
Can electrons quantum tunnel out of a wire and fly away, being effectively removed from the electonic current, resulting in a (localized) loss of energy?
Or is the amount of electrons in a wire just so big and the outermost layer where electrons could be lost so thin, that the effect is pretty much undetectable?

Actually, the issue here is that for tunneling, you need another nearby energy well to tunnel into; if the energy is all uphill, you won't ever see tunneling. I'm no expert on solid state physics, but my understanding is that the attractive forces between the metal atoms and the electrons make it so that their energy in the metal is lower than their energy in vacuum, so the electron just sees an infinite wall it won't ever tunnel through outside of the wire.

On the other hand, if you had another wire nearby, or if you injected energy into the system to raise the electron's energy, then you would see quantum tunneling effects because you get those nearby wells you need to observe it. Speaking of which, the energy you need to put in to get a metal to eject an electron is called the work function (http://en.wikipedia.org/wiki/Work_function).

Asta Kask
2014-02-25, 12:25 PM
The energy you need to get someone to post in a thread is called the lurk function.

Kato
2014-02-25, 02:45 PM
That's a bad attitude. Chemistry works because of quantum effects. Electronics work because of quantum effects. Quantum effects everyday life.

I guess Jay was more referring to the improbably low odds quantum phenomenon which have no equivalent in macro science phenomenons. Yeah, quantum effects make everyday life work but those are the ones that happen with odds of about 100%, quantum tunnelling (and probably a dozen other I forget right now) don't, and using them as explanations for stuff like "If I run into a wall and all my particles spontaneously tunnel through it I will pass through the wall" are things that will give you a headache, or worse. (Then again, wasn't quantum tunnelling what kept the sun working? Or was this one of the dismissed arguments?)

shawnhcorey
2014-02-25, 02:55 PM
Yes but you can't tell what quantum effect may also be effective at the macro level without detailed knowledge of the effect. You can't come up with a general rule-of-thumb. Each quantum effect has to be considered individually.

Jay R
2014-02-25, 03:54 PM
Simple answer: Quantum effects are usually subatomic. Don't try to apply quantum mechanics on a macro level, or you'll eventually wind up wondering why a living cat cannot observe itself inside the box.That's a bad attitude. Chemistry works because of quantum effects. Electronics work because of quantum effects. Quantum effects everyday life.

Chemistry works because of quantum effects on the subatomic level. Electronics works because of quantum effects on the subatomic level. Quantum effects everyday life because of what it does on the subatomic level.

I repeat: Don't try to apply those effects on a macro level.


Yes but you can't tell what quantum effect may also be effective at the macro level without detailed knowledge of the effect. You can't come up with a general rule-of-thumb. Each quantum effect has to be considered individually.

Hence the word "usually" on my quote above.

Razanir
2014-02-25, 04:02 PM
I actually have a question about statistics:

So a statistic is complete if E(g(T))=0 -> g(T)=0. That's by definition. And I know that if a statistic is complete, then the distribution of T is unique for any value of the parameters. My question is, is this a biconditional? Can I show that the distribution is unique for any value of the parameters, and say it's therefore complete?

Update: Never mind, I found a counterexample.

shawnhcorey
2014-02-25, 04:16 PM
Chemistry works because of quantum effects on the subatomic level. Electronics works because of quantum effects on the subatomic level. Quantum effects everyday life because of what it does on the subatomic level.

That's like saying since gravity is caused by protons, neutrons, and electrons, it doesn't effect us at the macro level.

Knaight
2014-02-25, 04:18 PM
Chemistry works because of quantum effects on the subatomic level.

Kind of. Chemistry works because of quantum effects on subatomic particles, with the wave behavior of electrons being absolutely key to what is going on. However, quite a bit of these quantum effects are essentially on the molecular level. Take resonance - that involves the wave structure of electrons being distributed over several atoms, and possibly entire fairly large monomers*. Similarly molecular shapes are dictated largely by quantum effects involving the interactions of electron waves all over a molecule, with particular importance being other atoms two bonds away. Then there is the matter of actual chemical reactions, which involves this behavior between two (or very rarely three) molecules involved in a collision. To some extent, the quantum effects are at the intramolecular and molecular levels in addition to the subatomic levels.

That's a minor quibble however. The intramolecular levels and molecular levels usually aren't anywhere near macroscopic levels. Even macroscopic molecules (e.g. larger polymers) have reactions that are way below the macroscopic level, and the distinct properties of macroscopic molecules generally have essentially nothing to do with quantum chemistry, and are either best examined with physics and statistical chemistry.

*Aromatic systems involving numerous carbons are the obvious case here, though even benzene is packing twelve atoms.

the_druid_droid
2014-02-25, 04:46 PM
Generally, the only macroscopic quantum effects we know of are things like superconductivity and superfluidity, where you get large cooperative effects in very cold collections of (possibly composite) bosons.

Even then, the question of what 'macroscopic' means in this case is a little fuzzy. We do have lots of particles sharing a quantum state, with long-range phase coherence, etc, but the properties of these systems (although really neat) aren't necessarily identical to those of an isolated microscopic quantum system blown up to human-familiar scales.

Jay R
2014-02-25, 11:46 PM
That's like saying since gravity is caused by protons, neutrons, and electrons, it doesn't effect us at the macro level.

Your analogy is not analogous. Gravity is classical mechanics, not quantum mechanics, precisely because quantum mechanics is defined as "the branch of mechanics that deals with the mathematical description of the motion and interaction of subatomic particles, incorporating the concepts of quantization of energy, wave-particle duality, the uncertainty principle, and the correspondence principle."

But let's get back to the subject at hand. Are you seriously arguing for tunneling effects large enough to penetrate an insulating sheath? Really?

Or to use the other example I gave, do you believe that an actual cat in a box would be both dead and alive until the box is opened?

factotum
2014-02-26, 03:19 AM
Or to use the other example I gave, do you believe that an actual cat in a box would be both dead and alive until the box is opened?

Depends--are there any air holes in this box? :smalltongue:

I actually agree with you, though--quantum effects may be the basis for things like chemistry and semiconductor electronics, but 99.999999% of the time you don't need to know that; you can combine two things in a chemical reaction and get an expected result without needing to know what's happening at the subatomic level. Otherwise all chemists would also need to be particle physicists, and that is most definitely not the case!

To go back to the original question: I suppose it's just about possible that you'd get some energy loss from an uninsulated wire due to quantum tunnelling effects, but the energy loss in question would be so microscopically small that it can be disregarded in most real-world applications. I think it's starting to become an issue in semiconductors because the individual feature size on modern CPUs is so small that things like this *can* have an effect, though.

Knaight
2014-02-26, 04:28 AM
I actually agree with you, though--quantum effects may be the basis for things like chemistry and semiconductor electronics, but 99.999999% of the time you don't need to know that; you can combine two things in a chemical reaction and get an expected result without needing to know what's happening at the subatomic level. Otherwise all chemists would also need to be particle physicists, and that is most definitely not the case!

You do need to understand things to some degree however. That electrons behave as a wave is pretty much core to understanding basic bonding, molecular shape, and the entire concept of polarity. You need that much even for the basic inorganic chemistry typical of 101 classes. You get more of this in regards to resonance and charge distribution over molecules, both of which are extremely important in organic chemistry. There's obviously chemistry that can be done without any of this, as the field predates quantum physics by a large margin, but the understanding of quantum effects is extremely helpful and a great deal of the basics of modern chemistry as practiced in industry** was developed after a quantum understanding.

That said, you don't need even the knowledge you'd see in an undergraduate physical chem textbook for most things*, let alone much of particle physics.

*For those unfamiliar with physical chemistry, there are a lot of very complicated equations, many of which are unsolvable and involve some rather...fun approximation techniques.

**Which obviously lags behind laboratory techniques to some extent.

Yora
2014-02-26, 07:39 AM
Since someone mentioned a cat...

What is meant by "observation" in quantum physics? It's never really said in anything I've read or watched, but isn't an "observation" simply any event in which micro-scale particles interact with a macro-scale object?
The event of the radiation making contact with the detector in Schroedingers box is already an "observation" and the wave function has collapsed, isn't it?
Now if the box were actually perfectly sealed and insulated, we would not be able to make a prediction if the detector detected radiactive decay until we open it and look. But the dection would already have happened or not happened (and killed the cat) even before we look. Right?

--

This guy got a lot (http://www.youtube.com/watch?v=rFtYzVJcWyA) of really strange (http://www.youtube.com/watch?v=cvq8laPb498) and funny toys (http://www.youtube.com/watch?v=AoueExyXkWY).

Eldan
2014-02-26, 09:06 AM
Schrödinger's box is a silly example on purpose. But the way I understand it:

Whether or not the detector "observes" the decay or not does not really matter for this example. From outside the box, we can't really tell what the detector has observed. The important part is that we have two systems which are totally isolated from one another.

shawnhcorey
2014-02-26, 09:25 AM
Gravity is classical mechanics...

So, everything discovered before QM cannot possibly have quantum effects. You're being arbitrary. You're dividing physics into two categories based on historical accident.

And that's my point. Everything in our universe is based on QM and Relativity, not classical physics. Classical physics is an approximation and every time you use it, you must ask if it's applicable. Or does it give the wrong answer because it is the simplified version.

Jay R
2014-02-26, 10:55 AM
So, everything discovered before QM cannot possibly have quantum effects. You're being arbitrary. You're dividing physics into two categories based on historical accident.

That is simply false. The definition I gave clearly and unambiguous divided it on based on interaction of subatomic particles. And I didn't do it, by the way. The definition was in quotes, because it was a quotation. According to the oxford Dictionary, quantum mechanics is "the branch of mechanics that deals with the mathematical description of the motion and interaction of subatomic particles, incorporating the concepts of quantization of energy, wave-particle duality, the uncertainty principle, and the correspondence principle (http://www.oxforddictionaries.com/definition/english/quantum-mechanics)."

Pretending I said that everything discovered before QM can have no quantum effects is simply denying what I did say, and making up something else.


And that's my point. Everything in our universe is based on QM and Relativity, not classical physics. Classical physics is an approximation and every time you use it, you must ask if it's applicable. Or does it give the wrong answer because it is the simplified version.

Classical physics is based on the measurements made at very low speeds, on the macroscopic level, and can usually be applied in the environment it was based on. While it is an approximation, all measurements are approximations.

For instance, I weigh 192 pounds (representing a mass of about 87 kg). When I drive to school today, at 55 miles per hour, there will be a relativistic effect on my mass, and to a stationary observer, I would appear to have a mass of 87.0000000000003. But that 192 pounds is an estimate from a bathroom scale. The 87 kgs is also an estimate, since 192 pounds actually is equivalent to 87.0897 kgs. The additional relativistic weight of less than one trillionth of a KG is simply not measurable. It's far less than the change in my mass when I lose a few skin cells washing my hands. So while that relativistic effect is real, it is unmeasurable and irrelevant in that frame of reference.

It is occasionally necessary to take relativity into account, but only when dealing with incredibly precise measurements. GPS satellites actually have to account for both special and general relativity, but very few other human activities require it.

In fact (getting back to the topic of the forum, which you keep ignoring), there are energy losses in electric wires. The main ones are the Joule effect (resistance of the wire turning energy to heat), magnetic effects (energy dissipated into an electrical field), and dielectric effects (energy lost into the insulating material). The original question really asks if some electrons can tunnel past the material, thus avoiding the dielectric effect. The answer is that if so, it is not a measurable quantity, like the relativistic weight gain above.

factotum
2014-02-26, 11:37 AM
Schrödinger's box is a silly example on purpose.

Yes, it always puzzles me that people use a thought experiment designed to show how ridiculous some aspects of quantum theory are as support for those same weird parts of the theory!

shawnhcorey
2014-02-26, 11:59 AM
The definition I gave clearly and unambiguous divided it on based on interaction of subatomic particles. And I didn't do it, by the way. The definition was in quotes, because it was a quotation. According to the oxford Dictionary, quantum mechanics is "the branch of mechanics that deals with the mathematical description of the motion and interaction of subatomic particles, incorporating the concepts of quantization of energy, wave-particle duality, the uncertainty principle, and the correspondence principle (http://www.oxforddictionaries.com/definition/english/quantum-mechanics)."

Completely overlooking the fact that QM does have effects on the macro level.


Yes, it always puzzles me that people use a thought experiment designed to show how ridiculous some aspects of quantum theory are as support for those same weird parts of the theory!

It's not ridiculous. Consider the double-sided experiment (https://en.wikipedia.org/wiki/Double-slit_experiment). Until recently, it was thought to be impossible to determine the path the particle takes. But they have measured the time it took and calculate the distance it travelled. And for even particle, the distance matches exactly the distance it would take to travel through of of the slits, a 50-50 chance. But it's only 50-50 because the source is always placed equal distance from both slits.

So, what really happens? The particle encounters the slits and becomes two superposition ones. They are also entangled with each other. This entanglement means they interfere with each other as waves. When the particle is measured, the superposition collapses and they spontaneously disentangle.

So, what does this have to do with the macro world? Air is a massive multiple-slit apparatus. When light passes through air, it forms a large entangled superposition of itself. That is why it can travel through air and it isn't scattered: it keeps information about itself in an entangled superposition. The universe is full of entangled superpositions. It is its normal state. Entangled superposition explains why there are red sunsets. QM effects everyday life. It is not just for the very small.

the_druid_droid
2014-02-26, 12:05 PM
Since someone mentioned a cat...

What is meant by "observation" in quantum physics? It's never really said in anything I've read or watched, but isn't an "observation" simply any event in which micro-scale particles interact with a macro-scale object?
The event of the radiation making contact with the detector in Schroedingers box is already an "observation" and the wave function has collapsed, isn't it?
Now if the box were actually perfectly sealed and insulated, we would not be able to make a prediction if the detector detected radiactive decay until we open it and look. But the dection would already have happened or not happened (and killed the cat) even before we look. Right?


This is actually a fairly tricky question, in part because it boils down to which interpretation of quantum mechanics you believe in. In particular, the original Copenhagen school didn't try to go much beyond the fact that measurements correspond to operators in the math and that they're things we can do experimentally, although we have to be careful in figuring out which observation we're actually making at times.

Later interpretations, like decoherence or many-worlds, take different viewpoints on the whole thing. Wikipedia (http://en.wikipedia.org/wiki/Interpretations_of_quantum_mechanics) has a pretty decent summary of the various theories.

EDIT: @shawnhcorey: What? Red sunsets are manifestly because light gets scattered in the atmosphere. There isn't some sort of weird entanglement going on; it's Rayleigh scattering (http://en.wikipedia.org/wiki/Raleigh_scattering)...

Jay R
2014-02-26, 01:13 PM
Completely overlooking the fact that QM does have effects on the macro level.

This is simply untrue. The first post of mine that you objected to said, "Quantum effects are usually subatomic." The word "usually" was there. No fair reading of my words on this thread can possibly lead to the conclusion that I overlooked effects on the macro level. Please stop telling this falsehood about me.

I haven't specifically talked about the macro effects caused by quantization, wave/particle duality, etc., because as far as I know, it is not germane to the question about wires.

shawnhcorey, you seem to disagree, but you have not yet provided any information. Do you have a specific example of a quantum mechanical effect that has an effect on the macro level regarding the tunnel effect causing a loss of power in a wire, or are you just determined to derail the thread?

I will once again try to drag the thread back to the actual question we were asked. It's theoretically possible that some electrons might get past the insulation by quantum tunneling, but quantum tunneling usually takes place at the level of 3 nanometers or less. I would expect the insulation on a wire to be thicker by a factor on the order of ten thousand or more. I have no knowledge of any experiment showing tunneling at that distance. Since shawnhcorey seems insistent that we must consider macro-level effects for this problem, perhaps he can cite an experiment that shows quantum tunneling at that distance.

Even if it can be shown to have occurred, any power loss from quantum tunneling is completely swamped by the power losses that the industry measures and deals with regularly - the Joule effect, magnetic fields, and the dielectric effect.

Yora
2014-02-26, 01:55 PM
EDIT: @shawnhcorey: What? Red sunsets are manifestly because light gets scattered in the atmosphere. There isn't some sort of weird entanglement going on; it's Rayleigh scattering (http://en.wikipedia.org/wiki/Raleigh_scattering)...

Speaking of which:
Is there a connection between the atmosphere being blue in sunlight, and it being able to filter out ultraviolet radiation but not so much infrared? (Since UV is basically super-blue.)

the_druid_droid
2014-02-26, 02:38 PM
Speaking of which:
Is there a connection between the atmosphere being blue in sunlight, and it being able to filter out ultraviolet radiation but not so much infrared? (Since UV is basically super-blue.)

While it's true that higher-frequency light (like the blue/violet end of the spectrum and UV rays) is more scattered, the actual filtering has more to do with the upper atmosphere's spectral absorption region. Specifically, ozone molecules in the stratosphere absorb UV radiation and then dissociate into an oxygen molecule and an O radical which rapidly reacts with other atmospheric components and converts the UV radiation into harmless thermal radiation (i.e. heat from the reaction).

Eldan
2014-02-26, 02:52 PM
Specifically, many molecules, among them Ozone, absorb some parts of UV, because the energy of UV rays is similar to their binding energies.

Yora
2014-02-26, 03:23 PM
So there are different effects at works for blue light and UV radiation?

shawnhcorey
2014-02-26, 03:23 PM
...are you just determined to derail the thread?

Read the subject; this thread is about science. Talking about science in a science thread is not derailing it.

Eldan
2014-02-26, 04:00 PM
So there are different effects at works for blue light and UV radiation?

Yes. One is refraction, the other is absorption.

Knaight
2014-02-26, 06:00 PM
Specifically, many molecules, among them Ozone, absorb some parts of UV, because the energy of UV rays is similar to their binding energies.

It's probably worth expanding on this a bit. The short version is that there are a few big classes of absorption spectra:

Atmoic Spectra: This is the spectra associated with electrons moving between energy states on an atom. It generally involves few highly distinct spectral lines, though under some circumstances (e.g. magnets) you get close pairs. These are mostly IR, UV, and visible.

Molecular Spectra: This is basically atomic spectra spread over a molecule. It's essentially over the same part of the spectrum, but usually absorbs over patches of ranges.

Rotational Spectra: Rotational spectra is a microwave effect, which basically involves quantized changes in angular momentum. It's essentially restricted to gas phase as well.

Vibrational Spectra: Chemical bonds have some amount of energy in them which involves vibration - minor cyclical changes in bond length essentially*, with the actual distance between the close and far points corresponding to variations in the energy of the system. Again, this is quantized, and generally corresponds to IR.

NMR: The short version is that nuclear spin can be altered, with electron shielding. For some atoms (isotopes of hydrogen and carbon are used most, but selenium-77 also comes up) the radiation of spins switching back can be measured. This is largely a radio wave effect, and NMR is the technique used to examine it. MRI is the same thing, but it looks very specifically at hydrogen atoms on water molecules.

Atomic and Molecular spectra can both include UV. Where this gets particularly common is in molecules with resonance stabilization, of which Ozone is one.

*Though bond length is actually used to refer to the length at energy minimums, so the terminology use is a bit sloppy here.

the_druid_droid
2014-02-26, 06:30 PM
Well said, Knaight. One last point I'd make is that if you're willing to consider dissociative events, or photoelectron emission, quite a lot of molecules absorb in the far UV range pretty strongly. This is the reason that 'vacuum UV' gets its name; the hardest UV radiation will get eaten up by just about anything other than empty space, which then dissociates to various radical products and/or free electrons and converts the photon energy into some mix of translation, rotation, vibration, internal electronic excitation and lower energy photons.

My understanding is that one of the things which makes ozone a major player in UV absorption on Earth is that in an atmosphere with a significant O2 component, there exist nice reactive cycles between O3, O2, and O radicals which help maintain an equilibrium concentration of the former. I imagine its specific UV absorption spectrum is probably also relevant, but I don't know what that looks like offhand compared to solar UV output.

factotum
2014-02-27, 02:31 AM
EDIT: @shawnhcorey: What? Red sunsets are manifestly because light gets scattered in the atmosphere. There isn't some sort of weird entanglement going on; it's Rayleigh scattering (http://en.wikipedia.org/wiki/Raleigh_scattering)...

And the sky being blue is also a consequence of light being scattered in the atmosphere, for that matter.

shawnhcorey
2014-02-27, 08:47 AM
EDIT: @shawnhcorey: What? Red sunsets are manifestly because light gets scattered in the atmosphere. There isn't some sort of weird entanglement going on; it's Rayleigh scattering (http://en.wikipedia.org/wiki/Raleigh_scattering)...

No, I said air is transparent because of entangled superposition of photons. If there wasn't a mechanism to preserve the information, you would be unable to see through transparent media.

Grinner
2014-02-27, 10:26 AM
Since quantum physics appears to the topic of the day...

http://i.imgur.com/c7IKMec.png

I get that the photon source would be something like a laser pointer; half-silvered glass would be the beam splitter; and the detectors would be something like photoresistors.

Assuming this device can measure light quickly enough*, I'd like to do some kind of statistical analysis on the output. I don't get how it's "quantum", though.

*It probably can't, since the flow of electricity is slower than the speed of light, meaning any results would be inaccurate.

factotum
2014-02-27, 11:33 AM
I don't get how it's "quantum", though.


As far as I can see, the idea there is that you're firing one photon at a time through the splitter, and which side of the splitter it comes out of is determined by quantum effects and is thus pretty much random. That's why it's quantum.

shawnhcorey: Care to post a link proving that? I've certainly never heard of the like, and I can't find anything online suggesting that's what makes things transparent. As far as I know, the photon that hits your eye after its traversal through a transparent medium is exactly the same as the one that was emitted by the light source, not some quantum entangled duplicate of it.

the_druid_droid
2014-02-27, 12:43 PM
No, I said air is transparent because of entangled superposition of photons. If there wasn't a mechanism to preserve the information, you would be unable to see through transparent media.


shawnhcorey: Care to post a link proving that? I've certainly never heard of the like, and I can't find anything online suggesting that's what makes things transparent. As far as I know, the photon that hits your eye after its traversal through a transparent medium is exactly the same as the one that was emitted by the light source, not some quantum entangled duplicate of it.

I'm with factotum here; I'm skeptical in the absence of a citation. Particularly since the thing about entanglement is that in most cases it decays extremely quickly on contact with a thermal bath, which in this case would be the atmosphere itself...

EDIT: And because to generate entangled states in the first place, you usually have to do some pretty crazy stuff in the lab.

shawnhcorey
2014-02-27, 02:39 PM
I'm with factotum here; I'm skeptical in the absence of a citation. Particularly since the thing about entanglement is that in most cases it decays extremely quickly on contact with a thermal bath, which in this case would be the atmosphere itself...

No, a thermal bath would not cause disentanglement any more than a cool one. Disentanglement happens when you measure an entangled property.


EDIT: And because to generate entangled states in the first place, you usually have to do some pretty crazy stuff in the lab.

No, the double-slit experiment creates entangled particles.

the_druid_droid
2014-02-27, 05:21 PM
No, a thermal bath would not cause disentanglement any more than a cool one. Disentanglement happens when you measure an entangled property.



No, the double-slit experiment creates entangled particles.

Ok, cite something here or I'm gonna just drop this conversation because it's going nowhere.

Tirunedeth
2014-02-27, 07:21 PM
No, the double-slit experiment creates entangled particles.

No, it does not. Entangled states are explicitly a feature multiple-particle systems, while the double slit experiment is a single particle phenomenon. If you consider the case of many particles, then the double-slit experiment is just normal wave diffraction. It is only when particles are fired one at a time, or when a detector determines which slit a particle passed through, that quantum mechanics becomes directly involved. The former is plainly a single particle system, while the latter is not distinguished by interactions between the particles (although I do recall reading a hypothesis that the reason for wave function collapse had to do with entanglement; it was just an attempt at accounting for collapse, though, not something that has been confirmed, and I don't think entanglement with the detector was what you were talking about, anyways).

To elaborate a bit, here's a footnote from Introduction to Quantum Mechanics, Second Edition, by David J. Griffiths:

It is emphatically not true that every two-particle wave function is a product of two one-particle wave functions. There exist so-called entangled states that cannot be decomposed this way. However: If particle 1 is in state a and particle 2 is in stateb, then the two particle state is a product. I know what you're thinking: "How could particle 1 not be in some state, and particle 2 in some other state?" The classic example is the singlet spin configuration - I can't tell you the state of particle 1 by itself, because it is "entangled" (Schrodinger's lovely word) with the state of particle 2. If 2 is measured, and found to be spin up, then 1 is spin down, but if 2 is spin down, then 1 is spin up.

Essentially, the wave function of a set of n non-interacting particles can be composed of a linear combination of products of n single-particle wave functions. If multiple terms are non-zero, then the state is entangled, since measuring one particle's state causes the wave function to collapse for the entire system of n particles.

In the hopes of not letting the thread get too stuck on this particular subject, here's something I've realized over the past semester or two: we call a lot of things vectors that aren't really all the same kind of thing. In particular, the position vector is not the same kind of thing as another vector representing something like velocity at that position. In Cartesian coordinates, they look similar, but if you transform to polar coordinates, you write the position vector as (r[/r], θ), but the other vector consists of components along the directions of increasing [i]r and θ. Both things are members of vector spaces, but are part of different vector spaces. Then you have the vectors that show up to represent states in quantum mechanics, or those used in statistics, which are also their own things. Hmm. That all sounds a little trivial when I say it like that, doesn't it?

shawnhcorey
2014-02-27, 07:36 PM
No, it does not. Entangled states are explicitly a feature multiple-particle systems, while the double slit experiment is a single particle phenomenon.

One of the parameters that determine the interference pattern is the distance between the slits. If the particle goes through one slit, how can it possible "know" how far away the other one is. And don't say the path of the particle can't be determined. My previous posts (if you had bothered) states that they timed the particles and determined that the length of the path it takes exactly matches the length from the source to one of the slits and on to the detector. How can it go through only one slit and still have an interference pattern?

Iruka
2014-02-27, 08:06 PM
No, I said air is transparent because of entangled superposition of photons. If there wasn't a mechanism to preserve the information, you would be unable to see through transparent media.

That stuff sounds very interesting. Where could I read read more about this entanglement stuff?

shawnhcorey
2014-02-27, 08:47 PM
That stuff sounds very interesting. Where could I read read more about this entanglement stuff?

Wikipedia is a good place to start.

shawnhcorey
2014-02-27, 08:48 PM
An article on astrophysics: How Astronomy Supports Evolution (http://briankoberlein.com/2014/02/27/astronomy-supports-evolution/)

Tirunedeth
2014-02-27, 09:43 PM
One of the parameters that determine the interference pattern is the distance between the slits. If the particle goes through one slit, how can it possible "know" how far away the other one is. And don't say the path of the particle can't be determined. My previous posts (if you had bothered) states that they timed the particles and determined that the length of the path it takes exactly matches the length from the source to one of the slits and on to the detector. How can it go through only one slit and still have an interference pattern?

Can you provide a link to a paper on the measurement of path length, or at least who "they" are, so that I can search for the paper myself? I'd like to know exactly what I'm discussing, particularly the details of the experiment and how the authors' explain their results, before attempting to explain what's going on.

In any case, in the standard explanation, one major point of the double slit experiment is that if the particle only passes through one slit (ascertained by placing a detector at one or both slits), the interference pattern no longer arises. You get the kind of pattern you would expect to see from firing grains of sand at a pair of slits, instead. Typically, however, the particle doesn't pass through just one slit; particle is a bit of a misnomer, as it is described by a wave function with some extent in space. That wave function passes through both slits, causing both slits to function as point sources, which results in a probability distribution identical to the intensity distribution for classical double-slit interference. Here (http://www.youtube.com/watch?v=jHyO0A7C86E) is a nice visualization of the process with a Gaussian incident wavepacket.

By the way, if the double slit duplicates photon, as in your explanation, how exactly is energy conserved? Further, shouldn't that mean that we should only ever observe two spots on the detector appearing at a time, not one?

Asta Kask
2014-02-27, 09:44 PM
One of the parameters that determine the interference pattern is the distance between the slits. If the particle goes through one slit, how can it possible "know" how far away the other one is. And don't say the path of the particle can't be determined. My previous posts (if you had bothered) states that they timed the particles and determined that the length of the path it takes exactly matches the length from the source to one of the slits and on to the detector. How can it go through only one slit and still have an interference pattern?

I thought it took every path at once?

shawnhcorey
2014-02-27, 10:16 PM
By the way, if the double slit duplicates photon, as in your explanation, how exactly is energy conserved? Further, shouldn't that mean that we should only ever observe two spots on the detector appearing at a time, not one?

The two particles are in superposition. Only one can be measured. When it is, the superposition collapses.

Knaight
2014-02-27, 11:23 PM
Well said, Knaight. One last point I'd make is that if you're willing to consider dissociative events, or photoelectron emission, quite a lot of molecules absorb in the far UV range pretty strongly. This is the reason that 'vacuum UV' gets its name; the hardest UV radiation will get eaten up by just about anything other than empty space, which then dissociates to various radical products and/or free electrons and converts the photon energy into some mix of translation, rotation, vibration, internal electronic excitation and lower energy photons.

That's a bit outside of the spectra I was mentioning, but it is very much worth noting. The production of free radicals in particular is something UV does quite a bit of, and that sucks down UV like nothing else, along with being really important chemically (in both reactions you actually want, and in avoiding reactions you don't).

warty goblin
2014-02-28, 12:51 AM
Today I wrote a Monte Carlo simulation to evaluate the MSE of different biased estimators of variance components. That is all.

factotum
2014-02-28, 02:31 AM
Wikipedia is a good place to start.

So your best attempt at posting a link to support your position is "look it up on Wikipedia"? Wow.

Eldan
2014-02-28, 02:47 AM
Today I wrote a Monte Carlo simulation to evaluate the MSE of different biased estimators of variance components. That is all.

How nice. I had to listen to a one hour talk about possible new methods for spacial regression. A mathematician talking to a room full of field ecologists. About ten minutes in, no one understood what was going on anymore, and we do a lot of statistics.

Asta Kask
2014-02-28, 05:55 AM
New question from the GaiaGirl:


What makes up 83% of blood.

We can safely exclude 'water' - as patients who are allergic to water (yes, believe it or not, there are documented cases - it's just like any other allergy! A simple Google Search will offer many cases) would be instantly dead. But they're not which disproves the theory that blood has water in it. The blood appears to be too oily to be composed of water so I'm wondering what's in between the blood cells. It makes sense that it is an oily liquid because this would prevent the cells from sticking to the walls of veins, arteries, etc.

I asked this question elsewhere but nobody knew. I'm wondering if anyone even tried to extract the liquid between red blood cells to see what it was composed of (smaller cells? ) I'm studying biology in college and when I got asked this question it stumped me I was also asked to name the components molecularly

My brain hurts. Can anyone help me with this?

Iruka
2014-02-28, 06:11 AM
New question from the GaiaGirl:



My brain hurts. Can anyone help me with this?

Aspirin and a good rest? Make sure to drink enough water. :smalltongue:

Or did you wonder how to answer her question?

noparlpf
2014-02-28, 06:31 AM
New question from the GaiaGirl:

My brain hurts. Can anyone help me with this?

...It's water mixed with a variety of proteins and salts, plus RBCs (and some immune cells, hormones, &c.). Yes, a few people do appear to be allergic to water, but from what I remember from those cases, it's very mild and might only have been with topical application (clearly drinking liquids never killed them).

TuggyNE
2014-02-28, 06:32 AM
New question from the GaiaGirl:



My brain hurts. Can anyone help me with this?

That might just be the stupidest thing I have ever heard, given that blood plasma is, in fact, 92% water, and doubtless the cells are also at least 70% water — just like every other cell in the body.

It is in fact actually possible to have a reaction to water that looks allergic. But this reaction is not technically an allergy and only occurs on the outer skin; drinking water does not cause problems (unless it hits the outer lip?), nor does the water already in the body. (The cause is not entirely certain, but may be due to extreme skin sensitivity to dissolved substances in the water, rather than the water itself.)

Iruka
2014-02-28, 07:04 AM
It is in fact actually possible to have a reaction to water that looks allergic. But this reaction is not technically an allergy and only occurs on the outer skin; drinking water does not cause problems (unless it hits the outer lip?), nor does the water already in the body.

There is at least one case (http://io9.com/you-can-be-allergic-to-water-512792897), where it also occurs in the oesophagus. Since the woman in question can drink diet coke, the H2O itself is most likely not the problem.

Eldan
2014-02-28, 07:07 AM
Hrm-hrm-hmmm. One, two, one, two. Yes. Hrm.

BUUUUUUULLLSHIIIIIIT.

Can't be an oily liquid. Cells are surrounded by lipipd membranes. If the liquid between them was oily, they would dissolve.

Asta Kask
2014-02-28, 07:25 AM
There is at least one case (http://io9.com/you-can-be-allergic-to-water-512792897), where it also occurs in the oesophagus. Since the woman in question can drink diet coke, the H2O itself is most likely not the problem.

Probably a psychosomatic reaction.


Hrm-hrm-hmmm. One, two, one, two. Yes. Hrm.

BUUUUUUULLLSHIIIIIIT.

Can't be an oily liquid. Cells are surrounded by lipipd membranes. If the liquid between them was oily, they would dissolve.

I've met people where I'd swear it was ethanol.

Iruka
2014-02-28, 07:37 AM
By the way, where can I find more of this person and the ... things they write? :smalltongue:

Asta Kask
2014-02-28, 07:43 AM
The blood thread. (http://sguforums.com/index.php?topic=48954.0)

And the Science and Skepticism forum on the same site. GaiaGirl93. This is doublethink on an epic scale.

shawnhcorey
2014-02-28, 08:08 AM
So your best attempt at posting a link to support your position is "look it up on Wikipedia"? Wow.

Why should I look up a link when the math is obvious?

Yora
2014-02-28, 08:22 AM
Regarding half-life: It describes an average for a very large quantity of atoms. But if you had just two, completely ididentical, atoms of a radioactive element, would it be possibly that one of them will decay within a tiny fraction of a second and the other one remain in it's current state for trillions of years?

Bringing statistics into it, does the fact that an atom of a radioactive element has already existed for n seconds change the probability that it will decay within the next second?

Or in other terms, do atoms "age" or do they simply decay at a completely random point in time? And is there even something like a "radioactive element", or do all elements have the chance to decay at any moment, with some just having much, much lower chances of it happening.

Iruka
2014-02-28, 08:35 AM
Why should I look up a link when the math is obvious?

Then how about a link that explains the math, especially for the double slit case? For the people who don't see the obviuosness in it?

shawnhcorey
2014-02-28, 08:58 AM
Then how about a link that explains the math, especially for the double slit case? For the people who don't see the obviuosness in it?

Why should I supply links for all of 19th and 20th century physics and math?

Eldan
2014-02-28, 09:32 AM
Why should I supply links for all of 19th and 20th century physics and math?

Because arguing that way is dickish, the math isn't obvious to non-physicists and finding an explanation isn't that easy?


And now, an anecdote on the glories of scientific work:

The Burnt-Out Light Bulb

Dramatis Personae: A Lab Intern, A Technician, An Electrician, Diverse Scientific Personel

So, a bulb burns out in the lighting arrangement for my binocular microscope. No big deal, I think. Go find a technician, they will know.

We screw it open, spend five minutes cleaning away dust together. Hm. This bulb looks kinda old.
Open a drawer. "No, we don't have any HLX64640 24V, 150W halogen bulbs."
We call supplies. No, they don't have any either.
What follows is a forty minute trip through every lab and microscopy room in the building, digging through ancient drawers and asking everyone in sight if they know any other place where some halogen bulbs might be hidden.
No, we only find 12V, or 120W or 6V or ones twice the size. Yay.
Finally, in an office that is currently empty, on the top floor, we open a cupboard and find a box of bulbs from 1986.

And this is what seems to cause the most delays in our work. 28 year old lightbulbs. Either that, or getting R statistics to work the way we know they "should" work.

warty goblin
2014-02-28, 10:57 AM
How nice. I had to listen to a one hour talk about possible new methods for spacial regression. A mathematician talking to a room full of field ecologists. About ten minutes in, no one understood what was going on anymore, and we do a lot of statistics.
Remember the rules of the math/stat presentation.
1) After five minutes anybody not at least a fourth year graduate student specializing in the topic has at best only a vague notion of what's going on.
2) After ten minutes nobody but three faculty members who have been there since 1983 and wrote the textbook on the subject has the foggiest idea of what's going on.
3) By fifteen minutes, the entire audience, bar those three, has given up, and is either thinking about pizza, not visibly falling asleep, or estimating how much overtime the presentation will go.
4) By thirty-five minutes, absolutely nobody except the presenter understands what is going on.
5) By 45 minutes, the presenter no longer knows what's going on.
6) The three faculty members ask a series of indecipherable questions, thus preventing the grad students from going out and ordering the pizza they've been fantasizing about for the last three quarters of an hour.
7) At least one of the three faculty members will spend five minutes arguing with the presenter over a misunderstanding so obvious that even the grad students, who can now almost taste the pizza, have figured it out.
8) Pizza.



And this is what seems to cause the most delays in our work. 28 year old lightbulbs. Either that, or getting R statistics to work the way we know they "should" work.

I spend an enormous quantity of my wrestling with R. The amazing thing is that they expect us to be able to write sophisticated code, but require no programming background.

Eldan
2014-02-28, 11:06 AM
Remember the rules of the math/stat presentation.
1) After five minutes anybody not at least a fourth year graduate student specializing in the topic has at best only a vague notion of what's going on.
2) After ten minutes nobody but three faculty members who have been there since 1983 and wrote the textbook on the subject has the foggiest idea of what's going on.
3) By fifteen minutes, the entire audience, bar those three, has given up, and is either thinking about pizza, not visibly falling asleep, or estimating how much overtime the presentation will go.
4) By thirty-five minutes, absolutely nobody except the presenter understands what is going on.
5) By 45 minutes, the presenter no longer knows what's going on.
6) The three faculty members ask a series of indecipherable questions, thus preventing the grad students from going out and ordering the pizza they've been fantasizing about for the last three quarters of an hour.
7) At least one of the three faculty members will spend five minutes arguing with the presenter over a misunderstanding so obvious that even the grad students, who can now almost taste the pizza, have figured it out.
8) Pizza.


THAT IS EXACTLY WHAT HAPPENED. Including points 5 and 7. At least said presenter even openly admitted point 5: "And here I found out my revolutionary new method for spatial regression doesn't work. I have no idea why, but I will still argue that it's better than the 30 working methods I compared it to." And point seven. God dammit was that misunderstanding stupid.

I never had a stat presentation before. It was before lunch break, too, and went fifteen minutes into overtime, so that half the food choices in the cafeteria were already sold out.

warty goblin
2014-02-28, 11:33 AM
THAT IS EXACTLY WHAT HAPPENED. Including points 5 and 7. At least said presenter even openly admitted point 5: "And here I found out my revolutionary new method for spatial regression doesn't work. I have no idea why, but I will still argue that it's better than the 30 working methods I compared it to." And point seven. God dammit was that misunderstanding stupid.

Heh, amateur. The way the truly savy math/stat presenter handles stage 5 is to say 'area of future interest.' This is Statisticianese for 'no goddamn idea, but I hope to get another three papers out of it.'


I never had a stat presentation before. It was before lunch break, too, and went fifteen minutes into overtime, so that half the food choices in the cafeteria were already sold out.
You lucky bastard. I have one of these every week, at 4:00 Monday afternoons. By this point I usually haven't eaten in seven or eight hours. These talks have given me fantasies involving pepperoni that count as at least three different sins simultaneously, believe me.

Two weeks ago I had three of these talks in one week.

Asta Kask
2014-02-28, 11:37 AM
The blood thread. (http://sguforums.com/index.php?topic=48954.0)

And the Science and Skepticism forum on the same site. GaiaGirl93. This is doublethink on an epic scale.

And now she's been banned.

the_druid_droid
2014-02-28, 11:55 AM
Regarding half-life: It describes an average for a very large quantity of atoms. But if you had just two, completely ididentical, atoms of a radioactive element, would it be possibly that one of them will decay within a tiny fraction of a second and the other one remain in it's current state for trillions of years?

Bringing statistics into it, does the fact that an atom of a radioactive element has already existed for n seconds change the probability that it will decay within the next second?

Or in other terms, do atoms "age" or do they simply decay at a completely random point in time? And is there even something like a "radioactive element", or do all elements have the chance to decay at any moment, with some just having much, much lower chances of it happening.

It's a little tangential to your question, perhaps, but there's an interesting phenomenon originating in quantum mechanics related to the decay (or non-decay) of unstable particles subject to continuous observation, the quantum Zeno effect (http://en.wikipedia.org/wiki/Quantum_Zeno_effect).

Oh, and for the record, shawnhcorey that bit in the article on:

'A "measurement" is equivalent to strongly coupling the quantum system to the noisy thermal environment for a brief period of time, and continuous strong coupling is equivalent to frequent "measurement".'

Is what I meant when I talked about an entangled system coupled to a thermal bath. So yes, that sort of thing does cause decoherence and dis-entangles particles which is why sunlight in the atmosphere definitely isn't in an entangled state, since said atmosphere is a giant collection of randomly moving degrees of freedom coupled to that photon.

Eldan
2014-02-28, 12:04 PM
You lucky bastard. I have one of these every week, at 4:00 Monday afternoons. By this point I usually haven't eaten in seven or eight hours. These talks have given me fantasies involving pepperoni that count as at least three different sins simultaneously, believe me.

Two weeks ago I had three of these talks in one week.

Our weekly talk is usually mostly genetics. Which makes more sense, as the group's official name is Ecological Genetics.

noparlpf
2014-02-28, 01:36 PM
The blood thread. (http://sguforums.com/index.php?topic=48954.0)

And the Science and Skepticism forum on the same site. GaiaGirl93. This is doublethink on an epic scale.

Thank you for that, it was pretty hilarious.

jonc83
2014-03-01, 01:12 AM
Regarding half-life: It describes an average for a very large quantity of atoms. But if you had just two, completely ididentical, atoms of a radioactive element, would it be possibly that one of them will decay within a tiny fraction of a second and the other one remain in it's current state for trillions of years?

Bringing statistics into it, does the fact that an atom of a radioactive element has already existed for n seconds change the probability that it will decay within the next second?

Or in other terms, do atoms "age" or do they simply decay at a completely random point in time? And is there even something like a "radioactive element", or do all elements have the chance to decay at any moment, with some just having much, much lower chances of it happening.

Half life describes the time taken for the activity of a sufficiently large sample of atoms to halve. It doesn't describe an individual atom at all-

You are quite right that an individual atom might decay quickly or take a long time. The probability doesn't change with age and it is impossible to predict when a particular atom might deca, so atoms do not age.

There are different stabilities of atoms of different elements and isotopes. Some are extremely stable, so very unstable. It may even be that protons themselves decay albeit with a very long half life, though there is little experimental evidence to prove this.

Ravens_cry
2014-03-01, 01:25 AM
I recently found out something I've long wondered but, until now, never knew the answer to: how we know a meteorite is from Mars when we have never gotten samples from Mars via probes or manned missions. It's near the end of this video (https://www.youtube.com/watch?feature=player_embedded&v=NyBmUGGMNYs), but it's not long and the rest is pretty neat too.

Yora
2014-03-01, 04:44 AM
I am having some trouble with Hawking Radiation.

The idea is, that pairs of particles spontaneously come into existance and right on the event horizon of a black hole, the anti-particle is more likely to get pulled into the black hole than the other particle.
But I can't find any information why anti-particles would be more likely to do that. Can anyone help with that?

Max™
2014-03-01, 06:07 AM
I am having some trouble with Hawking Radiation.

The idea is, that pairs of particles spontaneously come into existance and right on the event horizon of a black hole, the anti-particle is more likely to get pulled into the black hole than the other particle.
But I can't find any information why anti-particles would be more likely to do that. Can anyone help with that?

Photons are their own antiparticle.

Pair production of massive particles near an event horizon would probably be infrequent for anything but the lowest mass holes. A small enough hole would probably be hard to distinguish from an explosion, due to the weird ways that hawking radiation behaves with smaller and smaller event horizons, particularly once you start getting into subatomic holes, last I checked at least. That may be out of date.

As for the half life question, I recall a recent result setting a lower bound for proton half life at something like 10^40 years, with a probable value closer to 10^80 or 10^90 I think? 10^34 years or thereabouts with a 90% confidence from Super Kamiokande, certain aspects of physics prevent any value below 10^32 years being possible. I think the 10^40 value was for a certain stable isotope or something, not sure now. The 10^41 was an upper bound for proton lifetime if certain aspects of inflationary theory and baryogenesis are true. While 10^80 is the lifetime of some supermassive black holes like Sag A* as I recall.

Yora
2014-03-01, 09:27 AM
I was simply thinking of "ordinary" black holes just doing what they always do. No special case at all.
Why do anti-particles get pulled in while the other particle can escape?

The fact that a pair of particles can be seperated and prevented from annihilating each other if one ends up at just the right distance from an Event Horizon while the other one is still far enough away to escape doesn't seem to be anything unusual at all.
But that it's not a 50/50 chance which one of them gets pulled in and which one escapes seems very unexpected. Particles and anti-particles should be perfect mirror-images, but apparently they are not. What makes them behave differently?

Regarding proton decay, half-life would be a good benchmark for measurements. But I think the universe would probably start to behave very differently even when just one tenth or one hundreth of all protons have decayed. I would expect something like "the fabric of reality crumbling" to happen.
Though I am not sure if even that time would still be so far out that it doesn't matter anymore since all chemical activity would have already stopped because of the heat death.

But this brings up another question: If the half-life of a given sample is 50 years. Would that mean that it would take 75 years for 75% to decay, or 100 years for 75% to decay.
Given what I know about statistics, I think it's probably a stupid question (and the answer would obviously be 100 years), but my brain isn't at its optimum today.

Ravens_cry
2014-03-01, 09:44 AM
The latter. Half then half of the remainder. It's why it's so good for dating things (http://science.howstuffworks.com/environmental/earth/geology/carbon-14.htm), as you can tell how old something is by how much of whatever radioactive element is been measured is left in the sample compared to what would have been in original. No, it's not a stupid question. The only stupid question is when you aren't interested in the truth as such but are merely using the question as a weapon.

Jay R
2014-03-01, 10:13 AM
Regarding half-life: It describes an average for a very large quantity of atoms. But if you had just two, completely ididentical, atoms of a radioactive element, would it be possibly that one of them will decay within a tiny fraction of a second and the other one remain in it's current state for trillions of years?

Extremely unlikely, but yes, it's technically possible.


Bringing statistics into it, does the fact that an atom of a radioactive element has already existed for n seconds change the probability that it will decay within the next second?

No, it doesn't change the probability. Decay of a single atom has an exponential distribution, which is memoryless.


Or in other terms, do atoms "age" or do they simply decay at a completely random point in time? And is there even something like a "radioactive element", or do all elements have the chance to decay at any moment, with some just having much, much lower chances of it happening.

90 elements have isotopes believed to be stable. Some of the isotopes that are believed to be unstable for theoretical reasons have nonetheless never been seen to decay.

But really, how much difference is there between iron-56, which is stable, and Tellurium-128, which has a half-life (on the order of 10^24 years) orders of magnitude longer than the expected life of the universe?

Ravens_cry
2014-03-01, 10:36 AM
How do we know that's its half-life then?:smallconfused:
Again, might seem a stupid question, but I've always wondered.

Yora
2014-03-01, 10:41 AM
You don't have to sit with a stop watch and wait until 50% of your sample has decayed. If you can measure it precisely enough, you can wait for a month and see that 0.000000014% of your sample has decayed, you can extrapolate from that how long it would take until 50% are gone.

Ravens_cry
2014-03-01, 11:11 AM
Hmm, that makes sense, though I can see a lot of room for error with the longer half-lives, as when a particular atom decays is random. It just averages out for the bulk of the material over the half-life.

Yora
2014-03-01, 11:40 AM
But when you're talking about atoms, even a very tiny sample means a huge amount of individual atoms. And in statistics, you often get amazingly precise predictions from samples that seem rather small, IF you take the sample without as much bias as possible. And to watch for an atom decay or not decay is probably the situation with the least possibility for sample bias possible.
When you do statistics with people, there's a huge number of things that can change the result, depending on how you chose the people you are asking and the way you are asking them. With observing atoms, it's hardly possible to make such errors. Faulty equipment or a mislabled sample probably being the only possible mistakes.

Another thing that often confuses me is the Weak Force. Gravity and Electromagnetism are easy enough to understand, at least you can easily show what it does. Strong Force is also relatively easy to describe, since it's what counters the electromagnetic force and allows the protons in the nucleus of an atom to stick together instead of repelling each other.
But what does the weak force do? I've been looking everywhere and find a lot of information about W-bosons changing a proton into a neutron and a neutron into a photon. I understand that process, but what's the force part about this? The other three forces are all about pushing and pulling, but with the weak force there never seems to be any mention of particles being moved or held.

JCarter426
2014-03-01, 12:21 PM
It's hard to explain without getting too deep into quantum mechanics because the range is so short. So you really can't observe anything of the weak nuclear force in the macroscopic world so easily. Granted weak interaction is responsible for things like nuclear fusion and radioactive decay, but that happens on a scale that isn't quite familiar to us.

But one feature is that it changes the flavor of a quark. Flavor is a fundamental interaction like charge or mass. But the electromagnetic force cannot, for example, change the charge of any particle; it can make those particles attract or repel each other, but not change their properties. And that's what makes the weak nuclear force a bit crazy.

So to call it a force that changes particles into different kinds of particles is probably an oversimplification, but that's the best way I'm able to describe it.

warty goblin
2014-03-01, 02:14 PM
Hmm, that makes sense, though I can see a lot of room for error with the longer half-lives, as when a particular atom decays is random. It just averages out for the bulk of the material over the half-life.

This is a very basic statistics problem. Given a sufficiently large number of atoms, a completely reasonable model for the number that decay in any given time interval is a Poisson distribution with some mean. Then the waiting time between decay events will have exponential distribution with the same mean. Inference about simple means structures is really easy, as a general rule; the maximum likelihood estimators are usually even available in closed form. The half-life is a function of the mean, and since mles are invariant under transformation, once you have your data, you have a half-life estimate, with well understood asymptotics. One simple delta method later and you've got yourself a confidence interval for the half-life.

Alternatively you could do Bayes, but really, why would you here?

Ravens_cry
2014-03-01, 02:19 PM
OK, I admit, I haven't delved that far into maths yet. Thanks for the replies people. :)

Asta Kask
2014-03-01, 02:20 PM
"Use the Force, Luke."

"The Strong or the Weak?"

Ravens_cry
2014-03-01, 02:49 PM
"Use the Force, Luke."

"The Strong or the Weak?"
"I do not think you comprehend the gravity of the situation."

factotum
2014-03-01, 03:17 PM
I am having some trouble with Hawking Radiation.

The idea is, that pairs of particles spontaneously come into existance and right on the event horizon of a black hole, the anti-particle is more likely to get pulled into the black hole than the other particle.

I don't think either particle is more likely to get pulled in, are they? Whichever one escapes will either annihilate with a different particle or escape to form particle radiation.

Knaight
2014-03-01, 03:40 PM
This is a very basic statistics problem. Given a sufficiently large number of atoms, a completely reasonable model for the number that decay in any given time interval is a Poisson distribution with some mean.

As far as a sufficiently large number of atoms go - if you have a gram of an element, you could have about 6*10^24 atoms (hydrogen), or closer to 2*10^22 atom (Ununoctium), with anything in between. Using Yora's number of 0.000000014% decayed, you still get about 3*10^12 decayed atoms at minimum - which is way more than you need for statistical confidence. You could have something to the effect of 0.000000000000000014%, and still be fine, and that's with one gram.

Yora
2014-03-01, 04:19 PM
I don't think either particle is more likely to get pulled in, are they? Whichever one escapes will either annihilate with a different particle or escape to form particle radiation.
Common knowledge and sense would indeed make that seem the kind of situation one would usually expect. But from everything I've read, this isn't the case.
There is even the effect of black holes "evaporating", which I believe can even be observed in micro black holes. If a black hole has nothing to pull into its event horizon but those pairs of particles that spontaneously come into existance and disappear everywhere in the universe, it will consume more antimatter than matter in this way. And after sufficient time, this antimatter will cancel itself out with all the matter that was originally consumed by the black hole, while it still had stuff to pull into itself in its neaby surroundings.
Since black holes have, by definition, a really massive amount of mass, and the zone in which spontaneous particle pairs can be separated is extremely thin, this process should take a very, very, very long time.

It's an effect that is generally accepted to happen, but in our laymens understanding, it should not happen. And the question here is why this is the case. :smallbiggrin:

But one feature is that it changes the flavor of a quark. Flavor is a fundamental interaction like charge or mass. But the electromagnetic force cannot, for example, change the charge of any particle; it can make those particles attract or repel each other, but not change their properties. And that's what makes the weak nuclear force a bit crazy.

So to call it a force that changes particles into different kinds of particles is probably an oversimplification, but that's the best way I'm able to describe it.
The other three forces are, or at least seem, to be all interactions between two or more things. Gravity happening between any masses pulling at each other, electromagnetism taking place between charged particles, and the strong force holding the protons close together.
With the weak force, this does not seem to be the case. It's just neutrons spontaneously changing their composition. Or is the weak force an interaction between quarks tugging at each other in some way?

JCarter426
2014-03-01, 04:31 PM
As far as I understand it, the weak force is essentially unlike the other forces, no matter how you look at it. So there is no traditional push or pull... at least on a macroscopic level.

On a quantum level, I'm not qualified to explain it further, sorry. :smalltongue:

Jay R
2014-03-01, 05:25 PM
How do we know that's its half-life then?:smallconfused:
Again, might seem a stupid question, but I've always wondered.

Primarily by understanding how atoms decay, and studying the process. There are isotopes that we believe are unstable, with extremely long theoretical half-lives, that have never been observed to break down. The half-life of Tellurium-128 was measured, not by seeing the particles break down, by by measuring the quantities of its result, Xenon-128.

But I'm not an expert. I got some of the above by googling. My real field is operations research.

Knaight
2014-03-02, 02:24 AM
Primarily by understanding how atoms decay, and studying the process. There are isotopes that we believe are unstable, with extremely long theoretical half-lives, that have never been observed to break down. The half-life of Tellurium-128 was measured, not by seeing the particles break down, by by measuring the quantities of its result, Xenon-128.

This is pretty routine. It's actually even involved in things like the decay patterns of Uranium, as you can look at mixes of elements that emerge from it.

Max™
2014-03-02, 02:41 AM
Common knowledge and sense would indeed make that seem the kind of situation one would usually expect. But from everything I've read, this isn't the case.
There is even the effect of black holes "evaporating", which I believe can even be observed in micro black holes. If a black hole has nothing to pull into its event horizon but those pairs of particles that spontaneously come into existance and disappear everywhere in the universe, it will consume more antimatter than matter in this way. And after sufficient time, this antimatter will cancel itself out with all the matter that was originally consumed by the black hole, while it still had stuff to pull into itself in its neaby surroundings.
Whoa, gotta stop you there.

There is no way that I can think of to identify whether a black hole formed from a big lump of antimatter or from a big lump of matter. By definition if you could feed a hole a certain type of mass and have it react differently from another hole that would count as "hair", and they ain't s'posed ta have none. (http://en.wikipedia.org/wiki/No-hair_theorem)

Additionally, even IF a black hole was full of antimatter and you fed it a lump of matter and it tried to explode, it couldn't overcome it's own escape velocity anyways.

That is before we get into discussions like rotating time and space dimensions such that the future always "points" towards the singularity, and the interesting questions it raises for those who are fond of the handshake interpretation where antiparticules are just the time-reversed arrows on Feynman diagrams, which way do the arrows get reversed to when they all curve around towards the singularity?


Since black holes have, by definition, a really massive amount of mass, and the zone in which spontaneous particle pairs can be separated is extremely thin, this process should take a very, very, very long time.

It's an effect that is generally accepted to happen, but in our laymens understanding, it should not happen. And the question here is why this is the case. :smallbiggrin:
Oh, I think you got an odd idea about what takes place there-- which included the bit about the antiparticles preferentially falling in, I assume--spacetime is never quite empty, even if there are no particles around, there is a nonzero uh... I guess fizziness might make more sense than vacuum expectation value?

So instead of having a boring empty vacuum and setting up an experiment with the assumption that you will not detect any signals above a certain energy level, your apparatus picks up the occasional photon or whatnot.

There is a small but nonetheless positive chance that a pair of particles with a certain total energy will appear, briefly fly apart, then fall back together and annihilate back into nothingness.

The length of time available for a massive pair of particles is VERY short, as it is far less probable for nearly a Gev worth of electron+positron to zoot into existence and do ANYTHING before immediately vanishing.

Most of the "fizz" is stuff like neutrino/antineutrino or photon-photon pairs, which only "borrow" a small amount of energy and are relatively easy to get rid of again.

When some of that "fizz" pops up too close to an event horizon we have a problem, the pair was supposed to annihilate and give up the "borrowed" energy immediately, right?

Well, when one of the pair falls into the hole, there is a "deficit" that the vacuum isn't going to just ignore.

If there is no simple way to have the escaping particle vanish immediately, the energy it carries away has to have a real (rather than imaginary) value.

There is also the momentum gained by the infalling partner to account for!

So now we have a "loan" from nowhere that is supposed to pay nothing back to nobody, except the hole eats half of it, so it has to pay back the same amount it gained, except the escaping partner carries some momentum away as well.

The only way to reconcile this is to either throw away mass/energy conservation and just say heck with all of physics, replacing it with "a wizard did it lol", or accept that nothing can directly escape from a black hole... but it is possible to steal momentum, and thus mass-energy from one.

So the particle that is eaten could be written up in bookkeeping as the antipartner because obviously the escaping partner is now a real matter particle, but for the most part it is not going to be a common event to see an electron-positron pair pop up and last long enough to lose one or the other.

Even if this did happen, I expect the escaping positron would probably annihilate with something in the vicinity of the hole before long, giving a burst of gamma instead of the usual low energy photons making up most of Hawking Radiation.


The other three forces are, or at least seem, to be all interactions between two or more things. Gravity happening between any masses pulling at each other, electromagnetism taking place between charged particles, and the strong force holding the protons close together.
With the weak force, this does not seem to be the case. It's just neutrons spontaneously changing their composition. Or is the weak force an interaction between quarks tugging at each other in some way?

Uh... quarks tuggling at each other involves gluons, with their three colors (+/- red, green, and blue) and the different interactions there produce the various families of particles like protons and neutrons and mesons and all that noise.

Gluons interact with quarks AND other gluons, which means the fields between quarks begin to interact with themselves more strongly as you try to pull the quarks apart, and these interactions get energetic enough that beyond a certain point it is "easier" to pull a quark and antiquark pair out of the vacuum "fizz" I mentioned above, as it would take more energy to keep the quarks tied together than simply producing the new particles at that point.

It's all a lot more complicated than that, but suffice to say the strong force resists attempts to pull quarks apart, and a residual effect of the quark binding is involved in sticking protons and neutrons together inside atoms. Get a lump of quarks close enough to another lump of quarks and it turns out they're "sticky", even if they aren't directly bound together, hence the term "gluons" huh?

Now, the way that these lumps of quarks and protons/neutrons are stuck together, along with electrons and neutrinos (all the fermions) has a quirk that it involves something like uh... twists and curls in the forces sticking everything together, and the little heavy bits themselves have something called spin (i.e. you can tell that an electron is "pointing" one way or another, even though it doesn't have obvious features to identify this orientation by, this orientation behaves in ways which can kinda vaguely handwaving-y sound like the way tops and spinning balls behave, hence spin, again, waaaaayyy simplified and not totally accurate) and this twisted/spinning-ish nature shows up in an odd way.

If you watched a particle decay into other particles, and then watched the same event in a mirror, you'd expect there to be symmetry there and thus couldn't say that you were seeing a mirror image or an actual decay event.

Nature got drunk one night it turns out, and apparently certain types of decay favor one handedness over the other, allowing you to identify the real event from a mirror image.

Similarly we have the issue of "why matter and not antimatter", which turns out to be very closely related to this mirror symmetry (or parity) issue.

Altogether it seems that you can take a given particle decay event and replace all of the particles with antiparticles, and it will look like the mirror version.

This involves the W and Z bosons, again, because nature was drunk one night, and this weird handedness doesn't crop up much unless you get some nosy physicists reading nature's diary and they find out that kaons (quark/antiquark + strange quark/antiquark pairs) can turn into their own antiparticle, and this embarrassingly didn't happen the way it should have.

Again, nature, go home, you're drunk.

Other than that the weak force could just be thought of as "unwinding" the "twists" that help keep certain particles together, or "rewinding" those "twists" and turning one particle into another particle.

It's operating on a sorta unusual property of particles in a sorta unusual way and it only works over a very short range which means it must involve massive force carriers, but the main thing they do is stuff like beta decay, where one of the down quarks in a neutron turns into an up quark + a W boson, and the W decays into an antineutrino and an electron, meaning the electric charge and spin of the quark was altered, with the "leftover" charges winding up as the electron+antineutrino.

Oh, and there is another type of weak boson that only comes into play when neutrinos bump into other particles and give them a little speed boost, kicking an electron out of an atom without changing any of the spins or charges or whatnot required another boson that was neutral apparently.

Once again, too much grand unified tequila one night about 10^-12 seconds after the big bang, and now we've got all this silly twisty spinny particle stuff!

Oh, I forgot to mention one reason it is bad to think of fermion spin as being like actual spinning balls/tops: fermions are particles with half-integer spin.

How do you come up with something having "spin 1/2"?

Yeah, after the whole matter-antimatter fiasco and the mess with all those quarks getting stuck together, nature swore off the GUTequila for good.

factotum
2014-03-02, 02:43 AM
There is even the effect of black holes "evaporating", which I believe can even be observed in micro black holes.

That effect is nothing to do with the matter/antimatter ratio inside the black hole (what's in there isn't actually matter anyway--it's a singularity, far denser than even packed neutronium could be); when those matter/antimatter pairs pop into existence and do *not* immediately annihilate each other, the loss of energy has to come from somewhere, and the only place it can come from is the black hole itself.

Yora
2014-03-02, 03:35 AM
So it doesn't matter if the particle or the anti-particle falls into the black hole and the other one escapes? It's always a loss of energy for that region of space?

Asta Kask
2014-03-02, 05:33 AM
I guess so. Anything with mass (and antimatter particles have mass) has energy, so if it escapes it takes with it some mass.

Tirunedeth
2014-03-02, 10:41 AM
So it doesn't matter if the particle or the anti-particle falls into the black hole and the other one escapes? It's always a loss of energy for that region of space?

Precisely. Here's a quote from this (http://arxiv.org/abs/hep-th/9907001) paper (Hawking Radiation as Tunneling, Parikh & Wilczek) which sums the matter up nicely:


According to this picture, [Hawking] radiation arises by a process similar to electron-positron pair creation in a constant electric field. The idea is that the energy of a particle changes sign as it crosses the horizon, so that a pair created just inside or just outside the horizon can materialize with zero total energy, after one member of the pair has tunneled to the opposite side.

This is essentially just what factotum said, although I have some reservations about his comment about what is inside a black hole. I think it's better to simply note that there is no way of measuring what is inside the event horizon of a black hole, so any process such as Hawking radiation shouldn't depend on what is inside the event horizon.

Note that all of this assumes the black hole does not carry any charge. If it does, Hawking radiation should preferentially include particles of this charge (which will still include both matter and antimatter), which has the interesting effect of discharging the black hole.


Oh, I forgot to mention one reason it is bad to think of fermion spin as being like actual spinning balls/tops: fermions are particles with half-integer spin.

How do you come up with something having "spin 1/2"?
Is this part really so bad, though? All "spin 1/2" means is that the angular momentum is S = hbar * sqrt(1/2*(1/2 + 1)) = sqrt(3)/2 * hbar. Far more troubling is that, treating the electron as a uniform sphere with a radius equal to the classical electron radius, we obtain a speed of rotation at the surface of the electron of around ~1011 m/s, or ~300c. And it just gets worse as you consider smaller radii. Of course, that's a gross oversimplification of the situation, but I think it still shows the basic problem with treating "spin" as actually being due to spinning.

factotum
2014-03-02, 12:27 PM
although I have some reservations about his comment about what is inside a black hole.


If you can explain to me how ordinary matter can be denser than neutrons crushed together as close as they can go--which a black hole definitely is--then I'll accept I might be wrong about what's in there. :smallwink:

Ravens_cry
2014-03-02, 12:44 PM
The funny thing is a really big, massive black hole wouldn't crush you if you were inside the event horizon. See number 10 on this list (https://blogs.discovermagazine.com/badastronomy/2008/10/30/ten-things-you-dont-know-about-black-holes/). In fact, the more massive a black hole is, the less dense it has to be.

Tirunedeth
2014-03-02, 01:09 PM
If you can explain to me how ordinary matter can be denser than neutrons crushed together as close as they can go--which a black hole definitely is--then I'll accept I might be wrong about what's in there. :smallwink:

Oh, I wasn't trying to say that stuff inside black holes is ordinary matter. I just feel it's a bit inaccurate to talk about things inside a black hole at all, since the region inside the event horizon can't be observed. Not to mention that, in my understanding, time dilation causes any infalling matter to halt just above the event horizon (at least in the frame of a distant observer). Which, honestly, is a pretty minor quibble with what you said.

Yora
2014-03-02, 04:24 PM
Matter reaching the event horizon does not halt. The radiation-image of that matter does freeze and then simply starts to dim. The matter itself continues it's descent into the singularity inside the event horizon.

Tirunedeth
2014-03-02, 08:27 PM
Matter reaching the event horizon does not halt. The radiation-image of that matter does freeze and then simply starts to dim. The matter itself continues it's descent into the singularity inside the event horizon.

Hmm. You know, I was basing that interpretation off stuff from special relativity, where it's easy to pick and switch between reference frames, and things seen in any given inertial reference frame are an accurate description of reality (for that frame). I'm not sure how these ideas transfer to general relativity, so you may very well be right. It's a shame we haven't gotten to the actual GR in my general relativity class, in lieu of building up the mathematics needed to do GR (which, admittedly, is kind of important).

Max™
2014-03-03, 01:06 AM
Acceleration breaks the equivalence of frames, it's the easiest way to deal with the twin paradox, the twin which turned around had to accelerate to do so, and as such can determine that they were certainly the one in motion.

As for the spin, yeah, trying to deal with things like electrons being extended bodies has all sorts of issues, though you also get problems from point masses.

I stand by my statement that nature was a heavy drinker and the big bang was a wild kegger.

Tirunedeth
2014-03-03, 10:07 AM
Acceleration breaks the equivalence of frames, it's the easiest way to deal with the twin paradox, the twin which turned around had to accelerate to do so, and as such can determine that they were certainly the one in motion.

As for the spin, yeah, trying to deal with things like electrons being extended bodies has all sorts of issues, though you also get problems from point masses.

I stand by my statement that nature was a heavy drinker and the big bang was a wild kegger.

Well, even in accelerating frames, there still isn't any preferred frame, i.e. you can do physics in one frame and get the same result as doing physics in another frame and converting to the first frame (which, of course, is what makes the twin paradox so paradoxical in special relativity, until you realize that one twin switches frames in order to return to the other twin).

And, yeah. What's that saying about "fiction has to make sense; reality is under no such constraint?" Physics is like that (at least for certain values of "make sense;" the math works, after all).

the_druid_droid
2014-03-04, 02:25 AM
Well, even in accelerating frames, there still isn't any preferred frame, i.e. you can do physics in one frame and get the same result as doing physics in another frame and converting to the first frame (which, of course, is what makes the twin paradox so paradoxical in special relativity, until you realize that one twin switches frames in order to return to the other twin).

And, yeah. What's that saying about "fiction has to make sense; reality is under no such constraint?" Physics is like that (at least for certain values of "make sense;" the math works, after all).

I think the issue here is that while the object falling into the hole appears to pause and take infinite time to enter the hole from the perspective of a distant observer, in the object's own frame, it only requires a finite time to cross the event horizon.

I believe that's what Max is getting at above with the inequivalence of accelerated frames.

Asta Kask
2014-03-04, 07:10 AM
RISE! RISE FROM YOUR FROSTY GRAVE! (http://www.bbc.com/news/science-environment-26387276)

Fortunately it only infects amoeba, but yeah, these guys could have unleashed the next Black Plague. And as time goes on, and we exploit more of the tundra, there are more of those little darlings waiting down there.

Nice. Could make a good disaster movie.

Kato
2014-03-04, 08:11 AM
RISE! RISE FROM YOUR FROSTY GRAVE! (http://www.bbc.com/news/science-environment-26387276)

Fortunately it only infects amoeba, but yeah, these guys could have unleashed the next Black Plague. And as time goes on, and we exploit more of the tundra, there are more of those little darlings waiting down there.

Nice. Could make a good disaster movie.

Yeah, I just happened to read this bit as well... Except obviously the likelihood any random virus (no matter its size) is able to infect humans or anything else we care about is marginal... larger than the odds of the common cold being able to infect... say... Martians... but still small. It would make a fun disaster movie, though.

The fact of viruses growing that large is intriguing, though. I'm not that well versed in virology but I always depicted them as little more than a DNA strand a a protein shell. Which would make those a pretty huge DNA strand.

Asta Kask
2014-03-04, 08:24 AM
That is massive. E coli are between 500 nm and 2 000 nm.

noparlpf
2014-03-04, 08:57 AM
RISE! RISE FROM YOUR FROSTY GRAVE! (http://www.bbc.com/news/science-environment-26387276)

Fortunately it only infects amoeba, but yeah, these guys could have unleashed the next Black Plague. And as time goes on, and we exploit more of the tundra, there are more of those little darlings waiting down there.

Nice. Could make a good disaster movie.

Geez. That's a bloody huge virus. That's bigger than a lot of bacteria.

Wardog
2014-03-08, 02:17 PM
New question:

A lot of sci-fi features some form of "shield" that will block fast-moving/high-energy objects (like bullets), but allow the passage of slower objects, like people or docking space craft. Examples: Dune, Star Wars.

Is there any plausible mechanism for something like that?



Off-topic:

The only stupid question is when you aren't interested in the truth as such but are merely using the question as a weapon.

Thanks - that's something I've long been aware of, but never quite been able to put it in words.

factotum
2014-03-08, 03:55 PM
Is there any plausible mechanism for something like that?


We don't know of any realistic mechanism for a kinetic shield like that, but it stands to reason that you *would* allow slower items through because you'd need to for practical reasons--e.g. allowing shuttles to depart and arrive, in the case of ships.

Max™
2014-03-08, 06:26 PM
Hmmm, would you be opposed to trying out a device that involves zapping incoming objects with a beam of electrons and then trying to slow them down/deflect them with electromagnets?

I mean yeah, I can't say for sure that the system wouldn't produce some slight arcing, but a shield generator that shoots lightning is kinda awesome isn't it?

Oh, and make sure you don't try to do anything like incorporate a targeting laser into the electron beam and try to turn it into a FEL, and definitely don't design the electron accelerator in such a way as to make this an inevitable outcome...

Heck with it, I just want to see a shield generator that involves burning stuff with a FEL before it gets close unless it slows down...maybe even then.

Asta Kask
2014-03-08, 08:13 PM
Prairie dogs will call you fat. (http://www.treehugger.com/natural-sciences/researcher-decodes-praire-dog-language-discovers-theyve-been-calling-people-fat.html)

Cheeky little bastards! I'm just big-boned. :smallwink:

JCarter426
2014-03-08, 11:35 PM
New question:

A lot of sci-fi features some form of "shield" that will block fast-moving/high-energy objects (like bullets), but allow the passage of slower objects, like people or docking space craft. Examples: Dune, Star Wars.

Is there any plausible mechanism for something like that?
Assuming that "shields" in general are possible... then yeah, I don't see why not. Newton's Third Law. Every action has an equal and opposite reaction. If you have a force field that resists momentum, an object with greater momentum will encounter greater resistance.

The trouble is no such force field is known to exist... I'm not sure if the principle itself doesn't violate Newton's Third Law anyway.

Gray Mage
2014-03-09, 12:26 AM
New question:

A lot of sci-fi features some form of "shield" that will block fast-moving/high-energy objects (like bullets), but allow the passage of slower objects, like people or docking space craft. Examples: Dune, Star Wars.

Is there any plausible mechanism for something like that?

Well, it's not my area of expertise, but some non-Newtonion fluids are known to become thicker when a force is applied to them to the extent that it's possible to run on them (see here (http://www.youtube.com/watch?v=f2XQ97XHjVw) and here (http://www.youtube.com/watch?v=bLiNHqwgWaQ)). I immagine some kind of analogy could be made? Only for force fields somehow.

warty goblin
2014-03-09, 12:27 AM
Today I learned exactly how much I wish R was, in fact, Java. But no, instead of doing something simple with methods and objects, I spent the afternoon learning everything I never wanted to know about lapply.

Ravens_cry
2014-03-09, 12:50 AM
Well, it's not my area of expertise, but some non-Newtonion fluids are known to become thicker when a force is applied to them to the extent that it's possible to run on them (see here (http://www.youtube.com/watch?v=f2XQ97XHjVw) and here (http://www.youtube.com/watch?v=bLiNHqwgWaQ)). I immagine some kind of analogy could be made? Only for force fields somehow.
Fun stuff. You can run on it but you can't stand on it.
As far as force fields, besides the physics use of the term, there is plasma windows (http://en.wikipedia.org/wiki/Plasma_window), though they don't let the slow knife through.
I've pondered their potential use in closed cycle gas core nuclear rockets for the lightbulb in the nuclear lightbulb.

Kato
2014-03-09, 04:31 AM
Assuming that "shields" in general are possible... then yeah, I don't see why not. Newton's Third Law. Every action has an equal and opposite reaction. If you have a force field that resists momentum, an object with greater momentum will encounter greater resistance.


Um, pretty sure the average shuttle even if it's moving relatively slow still has about the same or more momentum than a bullet or even most rockets, because, you know, higher mass and all.

Let's say a bullet weighs roughly 10g. And moves at about 1000 m/s. (Both are by far not accurate but decent enough guesses) This means it's got a momentum of about 10 kg m/s. A rocket obviously is a good thing larger, let's say 10kg but on the other hand can't move as fast. But let's be generous and block it at 100 m/s. That's 1000 kg m/s. Quite a bit more momentum. But let's look at a (small) vessel. Even a tiny shuttle likely weighs around 1t=1000kg. (Very, very roughly the weight of a really light car) If we assume the thing is moving still at at least a slow walking pace of 1 m/s this turns out to be 1000 kg m/s as well, and we're pretty careful with our estimates on this one, it's likely ten times that.

So looking at that... maybe it's really the other way round.

So I guess the non-Newtonian fluid thing kind of makes sense except a force field isn't really... a fluid. (Non-Newtonian fluid are totally cool, though)


I think in most cases it's explained with more sciency stuff, like in Star Trek you can modulate your shields and if both vessels have the same shield frequency you can pass through the shield no problem. Or sensors scan the incoming object to determine whether it's a thread and then deactivate part of the shields. Or the writers just don't care and it's magic :smalltongue:

IthilanorStPete
2014-03-09, 10:12 AM
Why does it take more energy to detect higher-mass particles like the Higgs boson or top quark?

Asta Kask
2014-03-09, 10:16 AM
Einstein. E=mc2. So the more energy you put into the equation, the more mass you can get out.

noparlpf
2014-03-09, 10:26 AM
I'm researching wildcats for a natural history paper. But I'm running into some problems... (http://www.angelfire.com/dc/jinxx1/Wildcat/F4F_pt2.html)

Yora
2014-03-09, 10:38 AM
Yes, but there is a more comprehensive way to describe this correct answer.

The most fundamental and basic fact about physics is that everything has to come from somewhere, you can't make something out of nothing. Since energy equals mass (though not at a one to one ratio), only the total sum of all mass and energy before and after the change has to be equal.
In some process the particles that come out of a process have less mass than the particles that went into it and the rest of the mass is turned into energy. But also the reverse is true and you can get particles with more mass than the particles you started with, but this is only possible if you have added some extra energy. And obviously, when you create additional particles this way, a particle with more mass also needs more extra energy than a particle with small mass.

And the amount of energy that is equal to any give amount of mass is just massive. No pun intended. Because in E=mc², you don't just have E=m, but also multiply m by c², which is speed of light times speed of light. And so the amount of extra energy you need for additional mass is getting bigger and bigger very quickly.

Mrc.
2014-03-09, 10:50 AM
I thought it was E2=P2C2+M2C4? Vaguely recall this from physics, where P is the permeativity of free space. Could be wrong of course. Probably am!

Yora
2014-03-09, 11:00 AM
E=mc² is correct, but as Einstein also explained is that mass is affected by speed. But if you want to include the effect of speed, you can't just tack on a single additional modifier, but have to this much more complex equation.

E=mc² gets you the correct number if the mass is not moving at all, and I'm not sure if that's even actually possible in this universe or just a theoretical situation.

Which is a good question: Can anything actually be completely at rest and not moving in the post-big bang universe?

Asta Kask
2014-03-09, 11:40 AM
Moving compared to what?

Ravens_cry
2014-03-09, 01:19 PM
Well, no. Aside from the fact that everything is moving relative to something, it's impossible to reach absolute zero, so even on a subatomic level there is always movement. Also if a particle was absolutely still, we would know of its momentum AND its location with great certainty, which would violate Heisenberg's Uncertainty Principle I would think.

Asta Kask
2014-03-09, 01:24 PM
But this is General Relativity, where you get to choose your own reference system. I think the two may not be entirely compatible.

JCarter426
2014-03-09, 01:32 PM
Um, pretty sure the average shuttle even if it's moving relatively slow still has about the same or more momentum than a bullet or even most rockets, because, you know, higher mass and all.

Let's say a bullet weighs roughly 10g. And moves at about 1000 m/s. (Both are by far not accurate but decent enough guesses) This means it's got a momentum of about 10 kg m/s. A rocket obviously is a good thing larger, let's say 10kg but on the other hand can't move as fast. But let's be generous and block it at 100 m/s. That's 1000 kg m/s. Quite a bit more momentum. But let's look at a (small) vessel. Even a tiny shuttle likely weighs around 1t=1000kg. (Very, very roughly the weight of a really light car) If we assume the thing is moving still at at least a slow walking pace of 1 m/s this turns out to be 1000 kg m/s as well, and we're pretty careful with our estimates on this one, it's likely ten times that.

So looking at that... maybe it's really the other way round.

So I guess the non-Newtonian fluid thing kind of makes sense except a force field isn't really... a fluid. (Non-Newtonian fluid are totally cool, though)
Shuttle? Who said anything about a shuttle? :smallconfused:

Ravens_cry
2014-03-09, 01:42 PM
But this is General Relativity, where you get to choose your own reference system. I think the two may not be entirely compatible.

True, and that's one of the great puzzles of physics, trying to create a theory that unites the world of the Very Small with the Very Massive. When you get things that are both, like blackholes, the maths break down. I've read some articles that proposed it's impossible, but I don't buy that. It's only one Bulk after all.

Asta Kask
2014-03-09, 02:06 PM
True, and that's one of the great puzzles of physics, trying to create a theory that unites the world of the Very Small with the Very Massive. When you get things that are both, like blackholes, the maths break down. I've read some articles that proposed it's impossible, but I don't buy that. It's only one Bulk after all.

So what's your ideas on unifiying them? Do you think it's string theory, or loop quantum gravity, or Tipler's idea that we've already have a the theory and have had it for 40 years. Or are we still waiting for the great idea(s)?

Ravens_cry
2014-03-09, 03:01 PM
So what's your ideas on unifiying them? Do you think it's string theory, or loop quantum gravity, or Tipler's idea that we've already have a the theory and have had it for 40 years. Or are we still waiting for the great idea(s)?
I like to think the answer will be something new, but I honestly couldn't say that with any certainty beyond it being my opinion. Still, I like to think the best is yet to come.

Asta Kask
2014-03-09, 03:25 PM
I like to think the answer will be something new, but I honestly couldn't say that with any certainty beyond it being my opinion. Still, I like to think the best is yet to come.

Well, I asked for you're opinion so I'm happy.

Kato
2014-03-09, 03:45 PM
Shuttle? Who said anything about a shuttle? :smallconfused:
The original question included "(people or) docking space craft" which I took for e.g. a shuttle. Or an X-Wing. Or whatever you want to call it. Or did I miss something?


I thought it was E2=P2C2+M2C4? Vaguely recall this from physics, where P is the permeativity of free space. Could be wrong of course. Probably am!
Uhm, I think p in this is the momentum (or impulse? does it even matter? In German we say "Impuls" I'm not entirely sure on the translation(s)) which is why the actually speed of the particle somewhat matters. Obviously, c will most of the time be far bigger but... it's no permeativity from what I recall.

warty goblin
2014-03-09, 04:19 PM
Speaking of statistics, today I finished recoding my MCMC algorithm to sample from a zero-inflated Poisson model, where both the zero-inflation and mean parameters are themselves parametric functions of covariates. Test cases suggest a thousand-fold increase in efficiency compared to my previous implementation. Now it's on to the probability theory.

Mrc.
2014-03-09, 04:46 PM
Uhm, I think p in this is the momentum (or impulse? does it even matter? In German we say "Impuls" I'm not entirely sure on the translation(s)) which is why the actually speed of the particle somewhat matters. Obviously, c will most of the time be far bigger but... it's no permeativity from what I recall.

Yeah, that sounds right, because p=mv and if as was stated above, E=mc2 only works at zero velocity, p2c2 must equal 0. Thus E2=m2c4 which becomes E=mc2 when square rooted. Of course, I could be wrong again, I have noticed that physics seems to have that effect on me...

IthilanorStPete
2014-03-09, 10:37 PM
Yes, but there is a more comprehensive way to describe this correct answer.

The most fundamental and basic fact about physics is that everything has to come from somewhere, you can't make something out of nothing. Since energy equals mass (though not at a one to one ratio), only the total sum of all mass and energy before and after the change has to be equal.
In some process the particles that come out of a process have less mass than the particles that went into it and the rest of the mass is turned into energy. But also the reverse is true and you can get particles with more mass than the particles you started with, but this is only possible if you have added some extra energy. And obviously, when you create additional particles this way, a particle with more mass also needs more extra energy than a particle with small mass.

And the amount of energy that is equal to any give amount of mass is just massive. No pun intended. Because in E=mc², you don't just have E=m, but also multiply m by c², which is speed of light times speed of light. And so the amount of extra energy you need for additional mass is getting bigger and bigger very quickly.

But aren't things like the Higgs boson already "there", so to speak? Or is it created by a "reaction" of other elementary particles? If the latter, how does the Higgs mechanism work without involving Higgs bosons that could be detected? (My apologies if I'm overly simplifying things; I might just need to learn more physics to understand what's going on)

Max™
2014-03-10, 12:50 AM
But aren't things like the Higgs boson already "there", so to speak? Or is it created by a "reaction" of other elementary particles? If the latter, how does the Higgs mechanism work without involving Higgs bosons that could be detected? (My apologies if I'm overly simplifying things; I might just need to learn more physics to understand what's going on)

The Higgs bosons are degrees of freedom for a field with broken symmetry. Three of those degrees of freedom couple with the W and Z bosons giving them mass, the fourth has nothing else it can couple to so it has to be a new massive boson.

In order for this particle to be observed you need to raise the energies involved towards the conditions at which the original symmetry breaking took place.

Those conditions arose in the very early universe, extremely dense, extremely hot, when the universe cooled off sufficiently said symmetry was lost and you got the coupling of Higgs to weak force bosons and a Higgs field "left over". Particles which interact with this field obtain a property we call mass, fermions being most familiar of course.

The interaction with the weak force is to break a type of the symmetry involved, as the weak bosons would be massless if all of the weak force symmetries were preserved.

If you squish stuff back together hard enough you can get a situation where you can distinguish between the three degrees of freedom normally coupled to weak bosons and the final degree of freedom should be observable as a boson with a certain mass that is then called the Higgs Boson.

This state is unstable (balance a ball on top of a hill, it might stay for a short period, but it will most likely be found at the bottom of the hill at any point in the future) and decays in ways which can be predicted theoretically.

Confirmation of candidate particle behaviors and decay products was what the big fuss was over the Higgs in 2012 and 2013.

Asta Kask
2014-03-10, 12:52 AM
I think my brain just melted.

JCarter426
2014-03-10, 12:53 AM
The original question included "(people or) docking space craft" which I took for e.g. a shuttle. Or an X-Wing. Or whatever you want to call it. Or did I miss something?

Well, I wasn't commenting on the specifics, just the premise. But those are two completely different scales. In space you wouldn't be worrying about bullets traveling at 1000 m/s. You'd be worrying about bullets traveling at near the speed of light, 300 million m/s.

And I didn't want to bring this up, but now I can't resist... they don't actually have this kind in Star Wars. :smalltongue:

They just have different types of shields. Energy or ray shields block things like blasters, lightsabers, and radiation while allowing most matter to pass right through (I say most because it's commonly assumed that blasters and lightsabers are actually made of plasma). Particle fields block the physical stuff but not the other stuff. There must be different kinds of this one, because sometimes they keep everything out, but there is also the kind that can keep air and people inside while allowing ships and cargo to pass through. Possibly because a ship or crate acts as a Faraday cage? It allows anything inside to pass through too, after all. Or maybe there's some pseudoscientific field resonance like Star Trek. Anyway, none of them seem to be the stop fast stuff, allow slow stuff type.

Uhm, I think p in this is the momentum (or impulse? does it even matter? In German we say "Impuls" I'm not entirely sure on the translation(s)) which is why the actually speed of the particle somewhat matters. Obviously, c will most of the time be far bigger but... it's no permeativity from what I recall.
In English, impulse (J) is a change in momentum (p). Because why bother using letters that are in the words? :smalltongue:

Max™
2014-03-10, 12:54 AM
I think my brain just melted.
Trying to keep all that straight in my head wasn't very kind to my internal heat sinks, one good blast from a laser and I'm gonna go up in flames.

huttj509
2014-03-10, 03:10 AM
Well, I wasn't commenting on the specifics, just the premise. But those are two completely different scales. In space you wouldn't be worrying about bullets traveling at 1000 m/s. You'd be worrying about bullets traveling at near the speed of light, 300 million m/s.

And I didn't want to bring this up, but now I can't resist... they don't actually have this kind in Star Wars. :smalltongue:

They just have different types of shields. Energy or ray shields block things like blasters, lightsabers, and radiation while allowing most matter to pass right through (I say most because it's commonly assumed that blasters and lightsabers are actually made of plasma). Particle fields block the physical stuff but not the other stuff. There must be different kinds of this one, because sometimes they keep everything out, but there is also the kind that can keep air and people inside while allowing ships and cargo to pass through. Possibly because a ship or crate acts as a Faraday cage? It allows anything inside to pass through too, after all. Or maybe there's some pseudoscientific field resonance like Star Trek. Anyway, none of them seem to be the stop fast stuff, allow slow stuff type.

In English, impulse (J) is a change in momentum (p). Because why bother using letters that are in the words? :smalltongue:

You know, back in Undergrad, one time before physics class a few of us physics majors went through the entire English alphabet, and listed something typically indicated by each letter (such as B for magnetic field, m for mass, etc). Lowercase and Capital. M (and m) was already mass, and i (for impetus) indicates imaginary numbers, with I being current (though electrical engineering, last I heard, uses j for imaginary numbers since they use I and i more often for constant and variable currents. Caused a lot of confusion when the Physics majors were trying to help EEs with their homework.)

We didn't get around to doing a similar list for the Greek alphabet. Class was about to start.

Eldan
2014-03-10, 03:16 AM
Hmm. Greek Alphabet. Right away, I can't think of anything for alpha, beta or gamma. Not a good start :smalltongue:


Speaking of statistics, today I finished recoding my MCMC algorithm to sample from a zero-inflated Poisson model, where both the zero-inflation and mean parameters are themselves parametric functions of covariates. Test cases suggest a thousand-fold increase in efficiency compared to my previous implementation. Now it's on to the probability theory.

I think there's several nontrivial words in there that I've heard before :smalltongue:

Kato
2014-03-10, 06:10 AM
I think my brain just melted.
Yeah, same here... I guess I really should catch up on what that "god particle" is good for.


Well, I wasn't commenting on the specifics, just the premise. But those are two completely different scales. In space you wouldn't be worrying about bullets traveling at 1000 m/s. You'd be worrying about bullets traveling at near the speed of light, 300 million m/s.

While I'll admit bullets in space are likely a good bit faster than on earth accelerating massive objects close to c seems like a waste of resources. Though, most SciFi uses lasers anyway. Or they don't care :smallbiggrin:


In English, impulse (J) is a change in momentum (p). Because why bother using letters that are in the words? :smalltongue:
Yeah, English man, don't you dare complain. :smalltongue: We use m for "Gewicht" (though we also say "Masse" and I guess that's more precise but less common), v for "Geschwindigkeit", t for "Zeit", V for "Spannung", P for "Leistung", F for "Kraft"... English had a much more profound impact on scientific language than any other, or at least anything other than Latin. (At least we do call sodium Natrium... and potassium Kalium since we borrowed these from Latin. But apart from the the periodic table is also much more English than German. Oh, well...

warty goblin
2014-03-10, 10:18 AM
You know, back in Undergrad, one time before physics class a few of us physics majors went through the entire English alphabet, and listed something typically indicated by each letter (such as B for magnetic field, m for mass, etc). Lowercase and Capital. M (and m) was already mass, and i (for impetus) indicates imaginary numbers, with I being current (though electrical engineering, last I heard, uses j for imaginary numbers since they use I and i more often for constant and variable currents. Caused a lot of confusion when the Physics majors were trying to help EEs with their homework.)

We didn't get around to doing a similar list for the Greek alphabet. Class was about to start.
You think that's bad, in statistics we have a thing called a Gamma distribution, usually parameterized in alpha and beta. Thing is, there's two versions of this, both of which people will refer to as Gamma(alpha, beta), in spite of the fact that one person's beta is the reciprocal of another's.


Hmm. Greek Alphabet. Right away, I can't think of anything for alpha, beta or gamma. Not a good start :smalltongue:

I use gammas all the time, both lower and upper case. Lowercase is the letter of next resort for a regression parameter after the betas are exhausted. Upper case denotes the gamma function, which is common as houseflies in my line of work.


I think there's several nontrivial words in there that I've heard before :smalltongue:
It's not actually that complicated of a model. You've basically got a Poisson, except with an additional chance of being zero. Then hook up linear predictors to the extra zero probability and the Poisson mean using the obvious link functions, figure out a couple likelihoods, and you're good to go. With conjugate priors, most posteriors can be sampled from using a simple Gibbs algorithm, although I'm using Metropolis-Hastings for a couple.

The problem comes from making all the regression parameters random by group, when you have 190 groups. Damn model has 960 parameters in its full version, which makes doing a reasonably timely MCMC rather a bit of a coding challenge. Normally I'd use BUGS, but we aren't allowed for this assignment. So I did the whole bloody thing in R, which has all of the drawback of Java for this sort of thing, without the advantages of being strongly object oriented.

Asta Kask
2014-03-10, 10:35 AM
Poisson?

http://www.infovisual.info/02/img_fr/032%20Morphologie%20poisson%20osseux.jpg

huttj509
2014-03-10, 10:45 AM
Poisson?

http://www.infovisual.info/02/img_fr/032%20Morphologie%20poisson%20osseux.jpg

Meanwhile, the poison distribution involves tracking the assassination attempts across...nevermind.

warty goblin
2014-03-10, 10:48 AM
Poisson?

http://www.infovisual.info/02/img_fr/032%20Morphologie%20poisson%20osseux.jpg
Strangely enough, the Poisson distribution has more to with horses than fish.


Meanwhile, the poison distribution involves tracking the assassination attempts across...nevermind.
That's advanced Machiavellian geospatial statistics. Taking it automatically gets a person on all the best watchlists.

JCarter426
2014-03-10, 12:49 PM
Hmm. Greek Alphabet. Right away, I can't think of anything for alpha, beta or gamma. Not a good start :smalltongue:
Alpha for angular acceleration. I remember using gamma too... I think for magnetic flux. Don't remember ever using beta.

IthilanorStPete
2014-03-10, 01:41 PM
The Higgs bosons are degrees of freedom for a field with broken symmetry. Three of those degrees of freedom couple with the W and Z bosons giving them mass, the fourth has nothing else it can couple to so it has to be a new massive boson.

In order for this particle to be observed you need to raise the energies involved towards the conditions at which the original symmetry breaking took place.

Those conditions arose in the very early universe, extremely dense, extremely hot, when the universe cooled off sufficiently said symmetry was lost and you got the coupling of Higgs to weak force bosons and a Higgs field "left over". Particles which interact with this field obtain a property we call mass, fermions being most familiar of course.

The interaction with the weak force is to break a type of the symmetry involved, as the weak bosons would be massless if all of the weak force symmetries were preserved.

If you squish stuff back together hard enough you can get a situation where you can distinguish between the three degrees of freedom normally coupled to weak bosons and the final degree of freedom should be observable as a boson with a certain mass that is then called the Higgs Boson.

This state is unstable (balance a ball on top of a hill, it might stay for a short period, but it will most likely be found at the bottom of the hill at any point in the future) and decays in ways which can be predicted theoretically.

Confirmation of candidate particle behaviors and decay products was what the big fuss was over the Higgs in 2012 and 2013.

Okay. I don't understand exactly what's going on with symmetry breaking, but recreating the conditions that are hot enough that symmetry isn't broken makes sense to me. Thanks!

Asta Kask
2014-03-10, 02:21 PM
Alpha for angular acceleration. I remember using gamma too... I think for magnetic flux. Don't remember ever using beta.

Alpha is the fine structure constant, right?

Beta function, beta distribution, beta radiation...

Gamma function, gamma rays, etc.

I remember my statistics professor saying she had a hell of a time finding labels for her dissertation since everything was taken. I suggested futhark runes, but apparently that wouldn't do. :smalltongue:

huttj509
2014-03-10, 06:11 PM
Alpha is the fine structure constant, right?

Beta function, beta distribution, beta radiation...

Gamma function, gamma rays, etc.

I remember my statistics professor saying she had a hell of a time finding labels for her dissertation since everything was taken. I suggested futhark runes, but apparently that wouldn't do. :smalltongue:

It cracked me up when I learned mathematics starts using Hebrew when it deals with infinities.

Ravens_cry
2014-03-10, 07:11 PM
Alpha is the fine structure constant, right?

Beta function, beta distribution, beta radiation...

Gamma function, gamma rays, etc.

I remember my statistics professor saying she had a hell of a time finding labels for her dissertation since everything was taken. I suggested futhark runes, but apparently that wouldn't do. :smalltongue:
Don't forget alpha radiation, the least penetrative but most dangerous per dose if you actually manage to exposed, usually by swallowing or inhaling a source.

warty goblin
2014-03-10, 07:22 PM
It cracked me up when I learned mathematics starts using Hebrew when it deals with infinities.

Mathematicians will poach symbols from just about anywhere that's convenient. We use Arabic numbers, Roman numerals for marking parts of a multi-part statement, Phoenician letters in a plethora of fonts, Greek letters for everything not represented by 'X*', and, on occasion, actual stick diagrams. I've been tempted to start naming variables using Linear B characters.


*A habit that attains the apogee of confusing in statistics. X is a popular choice for a random variable, which can be observed at value x. Thus you end up with hastily scribbled notes filled with statements like P(X = x) = 0. This is when x isn't a fixed covariate value, groups of which are often written for convenience in a matrix denoted X.

Max™
2014-03-10, 11:39 PM
Okay. I don't understand exactly what's going on with symmetry breaking, but recreating the conditions that are hot enough that symmetry isn't broken makes sense to me. Thanks!

The symmetry breaking is what the ball on a hill is meant to illustrate.

Having a ball on top of a hill (assuming a perfectly spherical ball and the hill is a hemisphere as well here) then you could put an axis running straight down through the ball and hill, right?

If you rotated the ball+hill system around that axis there is no change, so there is a symmetry present which is not there for rotations perpendicular to that axis (or rotating the hill around the ball, which would leave the ball unchanged but the rest of the system would of course be rather different) and assuming the ball was carefully placed on the hill it might persist for a decent length of time.

The system is only meta-stable though, as the slightest input will cause it to rapidly move towards a different state, where the ball is now down at the bottom of the hill.

The ball accelerates in the process, converting potential energy into kinetic energy, which can then be dissipated as heat or impacts.

The system ends up in a state where it requires an input of energy to return it to the original more symmetrical arrangement, and said lack of symmetry can be seen by looking at what a rotation of the hill does now: the ball is no longer unchanged by this rotation while the hill is still symmetrical about that axis.

So now you have a spontaneous loss of symmetry for a portion of the system which is accompanied by a release of energy as the system or components thereof tend towards a lower energy/more stable state.

Now add a little extra portion to the hill and turn it into something resembling a sombrero, as long as the energy gained rolling down to the brim is not sufficient to roll up and over the edge, the ball will become trapped there instead of rolling all the way to the bottom.

This is a situation where another input of energy can liberate the ball, and assuming the remaining distance to the bottom of the hill is further than the sombrero-well was deep, you can apply a relatively small amount of energy in the right way and potentially get a very large return.

I think I just compared rolling a ball out of a hat down a hill to a nuclear weapon, so I'll stop there before I get any deeper into "apt-but-very-unusual-analogy-land" tonight.

JCarter426
2014-03-11, 02:19 AM
I just want to take this opportunity to point out that the above phenomenon is one of the reasons certain individuals were afraid the Large Hadron Collider could destroy the world. :smallsmile:

Alpha is the fine structure constant, right?

Beta function, beta distribution, beta radiation...

Gamma function, gamma rays, etc.

I remember my statistics professor saying she had a hell of a time finding labels for her dissertation since everything was taken. I suggested futhark runes, but apparently that wouldn't do. :smalltongue:
Of course, all the Greek letters get used up fairly easy in astronomy, too. But sadly I've never used any of those betas personally. I do use Greek letters in non-scientific situations, though. In writing. I use them to label my chapters.

The Extinguisher
2014-03-13, 01:27 AM
At least you guys have some context to figure out your variables, if your doing an electromagnetism thing, I is current. If your rotating rigid bodies, I is moment of inertia.
In pure math, you just use whatever letter is more convenient. If done homework sets that ran through the whole alphabet plus a handful of greek letters.

Question though. Does anyone know why physics and math mix up theta and phi in spherical co-ordinates. How does a small confusing change in convention happen like that?

JCarter426
2014-03-13, 05:37 AM
That's... a very good question. Theta is used for the polar angle in physics because of the right hand rule, but I have no idea what the reasoning for the other system is.

Knaight
2014-03-13, 04:11 PM
Question though. Does anyone know why physics and math mix up theta and phi in spherical co-ordinates. How does a small confusing change in convention happen like that?

I have no idea, but it happens all over the place - for instance, chem often handles angles in a clockwise fashion within polar and spherical coordinates, (e.g pi/2 when working with chem is instead 3pi/2 in standard coordinates).

Gray Mage
2014-03-13, 09:32 PM
At least you guys have some context to figure out your variables, if your doing an electromagnetism thing, I is current. If your rotating rigid bodies, I is moment of inertia.


What happens if the context is an eletromagnetic motor? :smalltongue:

Edit: Also, it was commented earlier, and yes, eletrical engineeres use j in place of i for imaginary numbers.

Eldan
2014-03-14, 11:55 AM
Mathematicians will poach symbols from just about anywhere that's convenient. We use Arabic numbers, Roman numerals for marking parts of a multi-part statement, Phoenician letters in a plethora of fonts, Greek letters for everything not represented by 'X*', and, on occasion, actual stick diagrams. I've been tempted to start naming variables using Linear B characters.

And now I imagine mathematicians some centuries in the future going full circle and resorting to Cuneiform since everything else is used up.

Kato
2014-03-14, 01:45 PM
And now I imagine mathematicians some centuries in the future going full circle and resorting to Cuneiform since everything else is used up.

Yeah, at some point they might even become desparate enough to use... numbers. (The horror)

Yora
2014-03-14, 03:20 PM
I've heard mathmaticians say that math doesn't have anything to do with numbers.

--

Are there any images that show what you would see if you were outside the solar system with no other light sources? Most space photos are color-enhanced or completely colorized based on other wavelength of emited radiation, so they don't really tell you how it actually looks.
I often read that space looks complete boring, but I've been to the countryside during clear nights on a couple of occasions, and even that looked kinda more impressive than "plain" space in most games and TV shows that don't add brightly colored nebulas to the scenes.
So I am curious how it would actually look like? I think it should probably be enough if a picture was taken from the far side of the moon while the Earth is between the Moon and the Sun. (Reflections from Mars or Jupiter should most probably be negible.)

TuggyNE
2014-03-14, 06:47 PM
I've heard mathmaticians say that math doesn't have anything to do with numbers.

It's like computer science, then, which has relatively little to do with computers. (The field is actually more concerned with how people work together efficiently in various ways. This is why the two hardest problem classes in CS are cache invalidation [how do we know when something is no longer useful?], naming [how do we communicate basic purpose and usage efficiently and reliably?], and preventing off-by-one errors [how do we remember to accurately consider every detail?]).

factotum
2014-03-15, 01:56 AM
Are there any images that show what you would see if you were outside the solar system with no other light sources?
.
.
.
So I am curious how it would actually look like? I think it should probably be enough if a picture was taken from the far side of the moon while the Earth is between the Moon and the Sun. (Reflections from Mars or Jupiter should most probably be negible.)

I'm a bit confused--first you say outside the Solar System, then you're talking about taking a shot just outside the Moon? That's two very different scales. Voyager 1 took a famous photo known as the "Pale Blue Dot"--that's the Earth as seen from a distance of six billion kilometres or thereabouts; it's the nearest thing I can think of to what you asked originally.

(Note that if you did somehow take a picture of the entire Solar System from outside it, you wouldn't actually see much--the Sun would be little more than a particularly bright star at that distance, and the planets would be pretty much invisible. That's how big the thing is).

Jay R
2014-03-15, 10:08 AM
I've heard mathmaticians say that math doesn't have anything to do with numbers.

When you've reduced the problem to just numbers, there's no math left, just arithmetic.

warty goblin
2014-03-15, 12:39 PM
When you've reduced the problem to just numbers, there's no math left, just arithmetic.

As we put it my department, the first time they ruined math by replacing all the numbers with letters. The second time the ruined it by putting numbers back in.

Max™
2014-03-16, 06:20 PM
I've heard mathmaticians say that math doesn't have anything to do with numbers.

--

Are there any images that show what you would see if you were outside the solar system with no other light sources? Most space photos are color-enhanced or completely colorized based on other wavelength of emited radiation, so they don't really tell you how it actually looks.
I often read that space looks complete boring, but I've been to the countryside during clear nights on a couple of occasions, and even that looked kinda more impressive than "plain" space in most games and TV shows that don't add brightly colored nebulas to the scenes.
So I am curious how it would actually look like? I think it should probably be enough if a picture was taken from the far side of the moon while the Earth is between the Moon and the Sun. (Reflections from Mars or Jupiter should most probably be negible.)

Well, it's not directly related to your question, but I made up a little game for myself in Celestia.

Basically I hit F8 several times until I'm going like a million times the speed of light, count to 10 while changing direction a few times, and then F1 to dump the speed.

Then I try to find my way back to the solar system with no grids or pointers turned on, only labels visible when you get close.

I think I lucked out the first time by finding M87 and orienting myself off of the jet which was enough to find M31 and the Milky Way, but that is where it got really hard.

I was able to find the more densely populated section of the galaxy easily enough (i.e. the far side doesn't have as many modeled stars and nebulae) but it's surprisingly difficult to turn "oh, there's Betelgeuse" into "and there's Sirius" and make the final step to "oh good, there's the Sun".

Think that last part between finding Betelgeuse and Sirius was a good 20 minutes of zooming around at a hundred light years per second or so.

Edit: Regarding the Sun itself, with the absolute magnitudes turned on right you can't see the Sun from much past Sirius at best, even Sirius doesn't show up beyond about 20 or 30 ly I don't think.

Luckily Betelgeuse and Rigel were visible from several hundred ly out and I was able to rotate them in my head until they were lined up properly relative to Sag A* and I could generally say "the Sun is a few hundred ly that way" from there.

Course, screaming into the solar system at multiple-light-year-per-second speeds is like "OH GOD WAS THAT A PLANET BRAKE WHERE DID IT GO" and then you gotta turn and find it all over again >.<...

Kato
2014-03-17, 10:02 AM
So I am curious how it would actually look like? I think it should probably be enough if a picture was taken from the far side of the moon while the Earth is between the Moon and the Sun. (Reflections from Mars or Jupiter should most probably be negible.)

I guess there are images though I couldn't tell you were to find them. Pretty sure Voyager might have made some (though, is it still sending?) but I don't think it's too impressive... but that's really just a random guess, based on the fact that you can get pretty easily images with little light polution even at the right time on earth (countryside at night with no moon) and you still don't see anything I would call amazing... And I think the atmosphere even helps a bit with making the few dots a bit more prominent. Otherwise it's really just a bunch of dots...

Max™
2014-03-17, 08:07 PM
So yeah: http://bicepkeck.org/b2_respap_arxiv_v1.pdf

Or, if you prefer: http://www.cfa.harvard.edu/news/2014-05

http://www.cfa.harvard.edu/sites/www.cfa.harvard.edu/files/images/pr/2014-05/1/hires.jpg

Gravity waves: O
Inflationary predictions confirmed: O


When asked to comment on the implications of this discovery, Harvard theorist Avi Loeb said, "This work offers new insights into some of our most basic questions: Why do we exist? How did the universe begin? These results are not only a smoking gun for inflation, they also tell us when inflation took place and how powerful the process was."

Kato
2014-03-18, 05:09 AM
So yeah: http://bicepkeck.org/b2_respap_arxiv_v1.pdf

Or, if you prefer: http://www.cfa.harvard.edu/news/2014-05

http://www.cfa.harvard.edu/sites/www.cfa.harvard.edu/files/images/pr/2014-05/1/hires.jpg

Gravity waves: O
Inflationary predictions confirmed: O

I know as a scientist I should be a bit more open but I'm always surprised what some people consider answers to fundamental questions. How does this answer "why do we exist?"? Except maybe on a really far fetched, how did the universe come to be as we know it, but that's not what I think of when I hear "why do we exist".
Also: Pretty image but without any context it really doesn't say anything. And even with context :smalltongue:

Don't get me wrong, it's amazing we found gravity waves and assuming someone didn't mess up the measurements this huge effect may lead to new interesting insights (though, the majority of the paper goes way over my head :smallredface:) but I still tend to be more skeptical when it comes to discoveries about the origin of the universe, fascinating as they may be.

factotum
2014-03-18, 07:20 AM
What this discovery does is provide proof for the generally-accepted theory. That might free up some people who are currently working on other theories to work on whatever comes next...it's probably not as remarkable as the media are making it out to be, though.

Max™
2014-03-18, 07:26 AM
The way the lines curl around is the signature of gravity waves they were hoping to find, as they have a handedness this provides an opportunity to look for them without needing to build several thousand miles long space based interferometers or any of the other "less mundane" methods proposed.

Like looking at the light through water which has been slightly disturbed, the pattern of ripples and twists in the image give a way of looking at the surface of the water itself, though it is rather difficult as it is damn near transparent.

Gravity waves and the CMBR are quite a bit more tenuous and thus needed long study periods in ideal conditions to be able to discern whether it was actually an effect in the distant universe rather than just atmospheric distortion or whatnot.

As they used the south pole for this to kill most of that difficulty, and we've gotten very good at looking through thick and moist air anyways, they were able to limit the cause of the swirls in the cmbr to being something very very far away.

The only thing which could have that sort of influence on that scale at that distance would be gravity waves, and anything closer would have a rather more detectable presence, hence the declaration that it's a gravity wave observation.

As for the inflationary model, we knew it fit the observations, but it was nice getting confirmation of predictions the model made about the specific distribution of said swirls and whatnot in the distant universe.

Kato
2014-03-18, 07:39 AM
The way the lines curl around is the signature of gravity waves they were hoping to find, as they have a handedness this provides an opportunity to look for them without needing to build several thousand miles long space based interferometers or any of the other "less mundane" methods proposed.
Oh, I get that but honestly, if you just had the image to go on, would you have any idea what it means? Heck, it doesn't even have a proper x-label and no legend or anything. Mind you, it's just from the article and these people I dare say don't care and think this is prettier than a proper graph but it's still... who is supposed to get anything out of the image?
It's really something I notice again and again even from respectable scientists, they sometimes can't make and label graphs properly... The papers fine in that regard, before someone thinks I talk specifically about that. Though, some of the error bars are pretty huge but at least they're being honest about it :smalltongue: (Not long ago I was talking to somebody about biologic papers and how they usually have uncertainties of plusminus 60% or such. I guess you can't blame them for their difficult experiments but it's still somewhat weird)

Max™
2014-03-18, 07:48 AM
Oh, I get that but honestly, if you just had the image to go on, would you have any idea what it means?
Well, me? Yes, but I've been keeping up with CMBR studies for pretty much all of my life, and seeing the structure there rather than a more chaotic mess is what made me stop and check the story out actually. Normally though, no, it isn't really something I'd expect to... pardon the pun... rise above the level of background noise to folks who hadn't previously had at least a passing interest in the subject.

Heck, it doesn't even have a proper x-label and no legend or anything. Mind you, it's just from the article and these people I dare say don't care and think this is prettier than a proper graph but it's still... who is supposed to get anything out of the image?
It's really something I notice again and again even from respectable scientists, they sometimes can't make and label graphs properly... The papers fine in that regard, before someone thinks I talk specifically about that. Though, some of the error bars are pretty huge but at least they're being honest about it :smalltongue: (Not long ago I was talking to somebody about biologic papers and how they usually have uncertainties of plusminus 60% or such. I guess you can't blame them for their difficult experiments but it's still somewhat weird)

Well, presenting the data runs the difficulties of having a graph which shows the expected outputs and observed results in a nice boring but honest set of lines with nicely labeled axes... or the more eyecatching image with the easily seen curls that make you look at it again just because it's neat looking.

Note that they quoted 7 sigma with one set of assumptions regarding foreground noise, 5+ sigma with a harsher cutoff, and precluded various other explanations for the observations at 2 to 2.9 sigma, which fits the "well, this is pretty damned interesting, but we haven't had a chance to even begin to do more but confirm that we were actually looking at something interesting, what does it look like to you all?" tone they had.

Naturally within a few minutes it winds up on some random regurgitated "science-y" headline site as "the fingerprint of creation" or whatever...:smallsigh:

Yora
2014-03-31, 08:51 AM
A science and tech forum? Wheee....:smallbiggrin:

Kato
2014-04-01, 01:30 PM
A science and tech forum? Wheee....:smallbiggrin:

My thoughts exactly :smallbiggrin: So this will for the time being be the random banter science thread?


So, short kick off question for biologists: what's the definition for s "sex"? Has it just become if it lays eggs it's female and everything else is male"? Because from what little I know about non-mammal biology the whole X and Y chromosome thing falls pretty flat with them, right? e.g. his crocodile gender depends on the temperature in the nest or how some fish change sex?

noparlpf
2014-04-01, 01:31 PM
My thoughts exactly :smallbiggrin: So this will for the time being be the random banter science thread?


So, short kick off question for biologists: what's the definition for s "sex"? Has it just become if it lays eggs it's female and everything else is male"? Because from what little I know about non-mammal biology the whole X and Y chromosome thing falls pretty flat with them, right? e.g. his crocodile gender depends on the temperature in the nest or how some fish change sex?

I think the definition is that a female produces eggs and a male produces sperm. I would have to look up a technical definition though to check.

Wardog
2014-04-01, 03:12 PM
Is there any plausible way for a planet to have Game of Thrones-style seasons? (I.e. summers and winters that last for an irregular number of years).

I haven't seen any official explanations.

It couldn't merely be that the planet (does it have a name?) has a very long orbit period, as that would just mean one very long year with correspondingly long-but-less-that-a-year seasons.

It would be possible with a binary solar system, but a) there is no mention of a second sun, and b) if it did, the seasons should be more predictable.

I suppose one alternative would be that the seasons have little to do with the orbit, and are more like an El Nino event. Although an El Nino that is more influential than the planet's orbit - is that plausible?

What about fluctuations in the sun's output - could that be enough to cause such changes?

noparlpf
2014-04-01, 03:21 PM
Is there any plausible way for a planet to have Game of Thrones-style seasons? (I.e. summers and winters that last for an irregular number of years).

I haven't seen any official explanations.

It couldn't merely be that the planet (does it have a name?) has a very long orbit period, as that would just mean one very long year with correspondingly long-but-less-that-a-year seasons.

It would be possible with a binary solar system, but a) there is no mention of a second sun, and b) if it did, the seasons should be more predictable.

I suppose one alternative would be that the seasons have little to do with the orbit, and are more like an El Nino event. Although an El Nino that is more influential than the planet's orbit - is that plausible?

What about fluctuations in the sun's output - could that be enough to cause such changes?

Seasons don't have to do with distance from the sun or which side of the sun the planet is on. Seasons have to do with the tilt of the planet. If a planet tilts back and forth with a period of eight years but orbits the sun with a period of one year, it'll have I believe two-year seasons. (I pulled those numbers out of my ear.)

warty goblin
2014-04-01, 03:23 PM
Is there any plausible way for a planet to have Game of Thrones-style seasons? (I.e. summers and winters that last for an irregular number of years).

I haven't seen any official explanations.

It couldn't merely be that the planet (does it have a name?) has a very long orbit period, as that would just mean one very long year with correspondingly long-but-less-that-a-year seasons.

It would be possible with a binary solar system, but a) there is no mention of a second sun, and b) if it did, the seasons should be more predictable.

I suppose one alternative would be that the seasons have little to do with the orbit, and are more like an El Nino event. Although an El Nino that is more influential than the planet's orbit - is that plausible?

What about fluctuations in the sun's output - could that be enough to cause such changes?

Seasons, at least on Earth, have very little to do with orbit per say, and much more to do with axial tilt*. If for some reason you had a planet that wobbled on its axis irregularly, one effect could well be fairly stochastic seeming seasons.


*For something with an extremely elliptical orbit that passes close to the sun, orbit is obviously much more responsible for temperature variation. Interestingly this appears at least twice that I know of in Martin's writings. Worlorn, the setting for Dying of the Light, is a rogue planet passing through the galactic disk, and at the time of the novel is rapidly moving too far away from the outermost stars to be habitable. A planet with extremely long winters and summers due to an eccentric orbit also appears as a setting for one of his short stories, and shows up as a reference in one of the Haviland Tough stories.

factotum
2014-04-01, 03:40 PM
I don't think there's any real-world physics that would explain that, no--if the sun's output were irregular enough to have this effect then chances are life would never evolve on the planet. Isn't the whole idea that the weird seasons in Game of Thrones are some sort of leftover magic effect from a long time ago, or, to put it another way, A Wizard Did It? :smallsmile:

noparlpf
2014-04-01, 04:50 PM
Note that the Earth is actually closer to the sun during wintertime (in the Northern hemisphere anyway, but that's where I live, so that's more important :smalltongue:) than it is during the summer. Axial tilt is what does it and a tilt with a longer period could yield seasons longer than a year.

MLai
2014-04-01, 11:22 PM
Excellent, I've found an appropriate GITP thread to post this query (I've read all 7 pages of this thread so far):

Johanan Raatz's Youtube videos... introductory examples here:
How Quantum Gravity Destroys Physicalism (http://www.youtube.com/watch?v=4NP4QmrbBww)
Quantum Physics Debunks Materialism (http://www.youtube.com/watch?v=4C5pq7W5yRM)

One of his more "detailed" videos here:
Physics Of The Hidden World (http://www.youtube.com/watch?v=Jzfj4R52Q6I)

So okay, I know he's a quantum mysticism proponent. But as a layman merely exposed to popular presentations of quantum mechanics and string theory etc (Nova channel, etc), I am not equipped to refute in my head what he's saying. And I want to (in my own head, I'm not interested in getting into a Youtube debate with him). So can those more familiar with these sciences just pick apart the foundations of his beliefs, in a way a layman like me can understand?

I do understand that his beliefs incorporate metaphysics as well, so my request may be too broad?

Max™
2014-04-02, 12:30 AM
Holographic descriptions of the universe have an unusual bit of evidence in their favor: the entropy of a collection of matter seems to depend on the surface area of a sphere containing the matter, rather than the volume of said sphere.

There are deep connections between works on black hole thermodynamics and information theory which led to certain conjectures which have found success as the basis for other work, one of the more well known being the http://en.wikipedia.org/wiki/AdS/CFT correspondence, which provides a way to examine things like the interior of a black hole through study of a mathematically related but more readily approached system in something called an anti-de Sitter space.

To put in a wrong but appropriate "lies for children" form, you're taking a complex problem in a spacetime with curvature like a sphere and rephrasing it in an easier form in a spacetime with curvature like a saddle.

An event horizon with a singularity in one spacetime looks like an expanding cloud of heated gas in the other, singularities are troublesome, expanding clouds of gas are pretty simple to work with by comparison.

...now, from all that we get the support for explanations of the universe as "it from bit", the Wheeler quote in the first video you linked from that guy.

The amount of entropy for a given amount of mass has an upper bound, this implies that there must be some fundamental level beyond which you can not subdivide matter, as each additional component adds a degree of freedom, and hence raises the entropy. Infinitely divisible components means infinite entropy, which means there are no infinitely divisible components.

Looking at the math for entropy of black holes/massive bodies and that for information, it looks like the only difference is one of units.

If you were to find that matter was composed of information there would no longer be a difference between the two and the math is nice and aesthetically pleasing.

Physicists like pretty math when it applies to the real world.

Jumping from that to "numbers are like, magic dude, like whoa" is absurd, if his point had any validity he would have something beyond vaguely stated import applied to coincidences or outright numerology.

________________________

As for the long seasons, I would mention that Mercury has a rather odd effect from the orbit-spin resonance where you can watch the sun come up, begin crossing the sky, stop and go the other way for a short period, then it continues as normal and sets: http://www.messenger-education.org/Interactives/ANIMATIONS/Day_On_Mercury/day_on_mercury.php

Naturally the lack of normal day-night cycles would be a giveaway in the books.

Beyond that if the winter and summer were consistent in lengths it could be due to orbital eccentricity rather easily.

As they seem to have fairly normal day night cycles and the year appears to be the same length as ours (had to check, never been interested enough in the series to know off hand) then the only answer beyond the official "it's magic" response Martin gave would probably require the axial precession to have a rather oddly specific resonance with the orbit.

If the axis were precessing quickly enough I suppose I could imagine a situation where it was quick enough to keep one pole more generally towards the star for years before swinging outwards for years of winter.

Unfortunately my brain starts screaming at me about the torque involved, there is no way I can picture that being a stable system over longer periods.

No clue how far back the stories extend discussions with the long winters and summers so that raises problems for non-magical explanations beyond "why haven't things evolved to adapt to this or died out" doesn't it?

Incidentally, I would think if the precession were opposite the direction the planet rotated/orbited--so if you viewed the system from above you would see the planet rotate counterclockwise, orbit counterclockwise, and the axis would precess in a clockwise fashion--then the various torques involved would slow down the summer part as the pole was cycling "inward" along the direction of the orbit, but that should cancel out for both hemispheres.

It may even be possible for a semi-stable solution with the precession taking place in such a fashion, but again, I don't see it lasting over geological periods of time.

MLai
2014-04-02, 03:45 AM
Holographic descriptions of the universe have an unusual bit of evidence in their favor: the entropy of a collection of matter seems to depend on the surface area of a sphere containing the matter, rather than the volume of said sphere.
There are deep connections between works on black hole thermodynamics and information theory which led to certain conjectures which have found success as the basis for other work, one of the more well known being the http://en.wikipedia.org/wiki/AdS/CFT correspondence, which provides a way to examine things like the interior of a black hole through study of a mathematically related but more readily approached system in something called an anti-de Sitter space.
To put in a wrong but appropriate "lies for children" form, you're taking a complex problem in a spacetime with curvature like a sphere and rephrasing it in an easier form in a spacetime with curvature like a saddle.
An event horizon with a singularity in one spacetime looks like an expanding cloud of heated gas in the other, singularities are troublesome, expanding clouds of gas are pretty simple to work with by comparison.
...now, from all that we get the support for explanations of the universe as "it from bit", the Wheeler quote in the first video you linked from that guy.
The amount of entropy for a given amount of mass has an upper bound, this implies that there must be some fundamental level beyond which you can not subdivide matter, as each additional component adds a degree of freedom, and hence raises the entropy. Infinitely divisible components means infinite entropy, which means there are no infinitely divisible components.
Looking at the math for entropy of black holes/massive bodies and that for information, it looks like the only difference is one of units.
If you were to find that matter was composed of information there would no longer be a difference between the two and the math is nice and aesthetically pleasing.
Physicists like pretty math when it applies to the real world.
Jumping from that to "numbers are like, magic dude, like whoa" is absurd, if his point had any validity he would have something beyond vaguely stated import applied to coincidences or outright numerology..
Well yes, ofc if he was for real (with some sort of competing theory, even if ultimately wrong in the eventual march of science), he'd be published rather than being some guy on Youtube. I'm actually looking at him as a reference for metaphysical technobabble. As in, if I wanted to write about pseudoscience as science in a sci-fi/fantasy story.

So when I'm asking "Can someone easily disprove this guy?" I'm actually asking "Is he blundering into any stupidly obvious pitfalls that I wouldn't want to follow into as a guy who wants to be known as Did The Research?"

So, if I were to summarize his major points without all the jargon from various disciplines, I think it would be:
(1) Holographic interpretation of universe, and all reality as composed of information, as you alluded to above.
(2) Consciousness drives reality. I'm pretty sure this idea was specifically disproven by the delayed-choice quantum eraser experiment and its variations, showing that the principle perplexing finding of the double-slit experiment occurs regardless whether a self-aware conscious human mind is observing. But this blogger is acting as if the experiment actually further supports the idea of quantum mind, or that wave function collapses due to interaction with consciousness. Major pitfall here?
(3) Cosmic consciousness, leading to the idea of the divine. I really don't care about this point as much. It's the previous 2 points which makes good sci-fi/ fantasy fodder.

So according to what you said, Max, this guy may be jumping from science into sci-fi, but it's not Hollywood-stupid sci-fi ala "aliens are invading Earth for the water"?

Max™
2014-04-02, 03:52 AM
Well, it's not as blatant for a layman, yeah, though it's probably more irritating for me, but I like my sci fi hard, like Baxter type stuff, so I couldn't sit through more of his videos without hurting my teeth gritting them.

As long as you just built on the parts you mentioned yourself and keep track of the pitfalls where he goes from "consciousness is important" to "therefore quantum jesus/etc" you should be fine.

aspi
2014-04-02, 04:59 AM
Is there any plausible way for a planet to have Game of Thrones-style seasons? (I.e. summers and winters that last for an irregular number of years).

I haven't seen any official explanations.

It couldn't merely be that the planet (does it have a name?) has a very long orbit period, as that would just mean one very long year with correspondingly long-but-less-that-a-year seasons.

It would be possible with a binary solar system, but a) there is no mention of a second sun, and b) if it did, the seasons should be more predictable.

I suppose one alternative would be that the seasons have little to do with the orbit, and are more like an El Nino event. Although an El Nino that is more influential than the planet's orbit - is that plausible?

What about fluctuations in the sun's output - could that be enough to cause such changes?
As others mentioned before, I don't think there's an easy astronomic explanation. However, there might be a meteorological one: Dust particles in the atmosphere that originate from even a single volcanic eruption can influence the climate dramatically. Take for example the Year Without a Summer (1816) that is believed to have been caused in part by the eruption of Mount Tambora. If the planet in question has an area with volcanic activity on a large scale, such particles might be released into the atmosphere in irregular intervals, thus causing longer winters. This is pretty much the nuclear winter scenario people were afraid of during the Cold War.

If we combine this with a low axial tilt of the planet (i.e. only a small difference between seasons), it creates the appearance of winters that last a lifetime, even though they are not actually winters in the common sense of the word.

Kato
2014-04-02, 05:41 AM
Is there any plausible way for a planet to have Game of Thrones-style seasons? (I.e. summers and winters that last for an irregular number of years).


As others mentioned, at least on earth it's really not the orbit that defines this but the tilt of the axis.
Obviously, there are a bunch of possible but unlikely explanations... e.g. for some reason (like lack of a moon) the tilt could be unstable and the planet could sway around (seemingly) irregularly which would result in different seasons... or the star could be going through much stronger phases of stringer/weaker activity. Or another, close, large planet could affect the planets tilt. Or it could work with certain eccentricity, I think, if it were to shift in an irregular pattern.
Likely, with sufficiently long studies, Westerosi would be able to figure out long term behavior if any of this was the case...

But most probable: A Wizard Did it.

Eldan
2014-04-02, 05:57 AM
The axial tilt of Earth changes too, by the way. It's on a "wobble" taking several tens of thousands of years, changing by about 1.5 degrees, from about 24° to about 22.5°. It has also been linked to ice ages by some theories, so there's some evidence behind that.
If Westeros' planet wobbled strongly and quickly (very quickly), that could explain a lot.

factotum
2014-04-02, 06:15 AM
If Westeros' planet wobbled strongly and quickly (very quickly), that could explain a lot.

A "wobble" like that is usually just as regular as normal seasons, though, so even if the axial tilt was changing rapidly enough to cause a difference (and that would be very, very fast indeed in planetary terms) you'd expect, say, 5 years of colder weather followed by 5 years of warmer. Wardog's post implied that summers and winters in Westeros are irregular, so they DON'T last a consistent amount of time like that.

noparlpf
2014-04-02, 06:40 AM
I dunno how Game of Thrones seasons work, but would it be possible for an irregular wobble such that the planet rocks, say, 15° one way but only 4° the other way? You'd have a short, mild season and a long, more severe season.

Eldan
2014-04-02, 06:46 AM
Westeros has a "Summer" and a "Winter", each of which last an irregular number of years. Up to decades. Though I'm not sure how they actually define "year" in Westeros, then.

The North is also remarked to have short "little winters" some years. So, maybe they have regular seasons too, they are just weak.

Earth has several cycles of change, I've noticed with some more research. The tilt of the axis changes, the perihelion changes, the time of the perihelion changes... and all these take different amounts of time.

So, for apparent irregularity, how about several cycles stacked on top of each other, potentially enough that a pattern really can't be noticed in a human timeframe?

MLai
2014-04-02, 07:01 AM
Well, it's not as blatant for a layman, yeah, though it's probably more irritating for me, but I like my sci fi hard, like Baxter type stuff, so I couldn't sit through more of his videos without hurting my teeth gritting them..
I get the same way watching creationism vs evolution videos, so I feel ya brah.
BTW thanks for the AdS/CFT correspondence wiki link. I don't know why but I can read that particular string theory page and actually understand it fully, which is not the case with almost every other ST/QM page on Wiki. I think this page had a particularly good editor who understands how to write clearly for (informed) laymen.

Max™
2014-04-02, 12:41 PM
It's also one of the less arcane aspects of stringy models, working your head around additional perpendicular directions which are curled up is a heck of a thing compared to "you can kinda turn a universe model inside out and it's easier to work with mathematically" I think.

Wardog
2014-04-02, 02:58 PM
Seasons don't have to do with distance from the sun or which side of the sun the planet is on. Seasons have to do with the tilt of the planet. If a planet tilts back and forth with a period of eight years but orbits the sun with a period of one year, it'll have I believe two-year seasons. (I pulled those numbers out of my ear.)

Ah yes, like a really short-period Milankovitch cycle. That could be a possibility.

And the apparent irregularity of the seasons could just be that the various components (tilt, precessions, eccentricity, etc), plus any meteorological or oceanographic feedback effects are too complex for a medieval society to distinguish.

Eldan
2014-04-02, 03:30 PM
Yup. I officially call it the "Wobble Stack" theory. :smalltongue:

Tanuki Tales
2014-04-08, 12:56 PM
This is basically an "Ask a Science question" thread, right?

Assuming so:

How fast would a sword need to be swung in order to superheat it to sufficiently sear flesh to the point its completely charred?

factotum
2014-04-08, 01:16 PM
I'm not sure that would even be possible--you'd have to accelerate the sword blade so fast that it would probably snap in half before it ever got close to hitting the enemy's flesh, not to mention that a sword which got so hot really wouldn't be much use as a sword again once it had cooled down due to the tempered metal reverting to something much more amorphous. Also, the faster the sword travelled the hotter it would have to get to char flesh (because it spends less time in contact with it), so there's a vicious circle to contend with as well.

Tanuki Tales
2014-04-08, 01:18 PM
I'm not sure that would even be possible--you'd have to accelerate the sword blade so fast that it would probably snap in half before it ever got close to hitting the enemy's flesh, not to mention that a sword which got so hot really wouldn't be much use as a sword again once it had cooled down due to the tempered metal reverting to something much more amorphous.

Obviously, for the sake of the question, the sword can survive the actual act.

Lord Torath
2014-04-08, 01:23 PM
Faster than the speed of sound, at any rate. Considering that the distance you're swinging through is roughly 6-10 feet (5 feet behind you to 5 feet in front of you), you're not going to have a lot of time for the sword to heat up.

The heat will be generated by the shockwave of the sword moving through the air faster than the air can transmit the information that it is coming. The faster your sword is moving, the stronger that shockwave will be, but that also means you will have less time for the sword to absorb that heat and become hot.

At Mach 1 (assuming standard temperature and pressure) sound travels through the air at about 1000 ft per second. That means your swing will need to take no longer than 0.01 seconds to break the sound barrier. Considering you're accelerating 4 to 10 lbs of steel from zero to 670 mph in 0.01 seconds, you're going to need just a wee bit of force.

Just off the top of my head, you'd probably need to be in the Hypersonic range (above Mach 5 if my memory serves correctly) to generate enough heat, but then you've only got 0.002 seconds or less for the heat transfer to take place. :smalleek: And that's for the sword to heat up. Assuming you're cutting through a foot of flesh, you've only got 0.0002 seconds to then transfer the heat from your sword to the target.

Unfortunately, it's been more years than I care to admit since my Compressible Flow classes :smallredface:. I'd need to dig into my books pretty hard to figure this out, if it's even possible to generate that kind of heat in that short amount of time.

Edit: It's probably worth noting that only the tiniest layer of the steel would have a chance to absorb the heat. Within a second of your swing, your sword would have dropped back to probably only 10-20 degrees hotter than it was before your swing as the heat diffuses through the rest of the sword.

I suppose a better way to approach this would be to calculate how much energy you need to completely char your target, then calculate how fast you'd need to be going to generate that amount of energy in the 3.5 meters you have available to swing your sword. I suspect it may be in hundreds or thousands of kilometers per second.

MLai
2014-04-08, 08:43 PM
Now that the Rouroni Kenshin question has been answered LOL, I have a more standard one.

If time is a dimension as in spacetime, then what the heck is a tesseract/hypercube? Which one is "true", and which one is purely mathematical/conceptual?

And if the hypercube is not "real", then what the heck is it? Is it extra dimensions or not? When they say "extra dimensions" in string theory, do then mean these extra dimensions as embodied by the hypercube, or something else entirely??

*brain hurts*

Knaight
2014-04-08, 09:09 PM
And if the hypercube is not "real", then what the heck is it? Is it extra dimensions or not? When they say "extra dimensions" in string theory, do then mean these extra dimensions as embodied by the hypercube, or something else entirely??

String theory gets a bit odd here, but I'll stick to the hypercube. Basically, it's a mathematical construct (just like lots of ideal shapes), which assumes four dimensions. It doesn't correspond to an actual object, but it is useful mathematically - e.g. a cube can be modeled as the surface of a hypercube, which lets you do some calculus you couldn't otherwise do, and the use of another variable that isn't a dimension (e.g. temperature) often involves similar math.

Grinner
2014-04-08, 09:16 PM
*snip*

As I understand it, a hypercube is a four dimensional cube.

To put it another way, lines are one dimensional things. A square is formed when lines meet at ninety degree angles, and a cube is formed when squares meet at ninety degree angles. So what is formed when cubes meet at ninety degree angles?

For a clearer and more entertaining explanation, I'm going to refer you to Flatland: A Romance of Many Dimensions (https://archive.org/details/flatlandromanceo00abbouoft).

Tanuki Tales
2014-04-08, 09:22 PM
Now that the Rouroni Kenshin question has been answered LOL, I have a more standard one.

Actually, it was concerning Wonder Woman beating a hydra. :smalltongue:

MLai
2014-04-08, 09:48 PM
I think what I'm trying to get to the bottom of, is this:
Which is the true "4th dimension", time or this property displayed by the hypercube?

Knaight
2014-04-08, 10:40 PM
I think what I'm trying to get to the bottom of, is this:
Which is the true "4th dimension", time or this property displayed by the hypercube?

It's not like there is a "true 4th dimension". "4th dimension" is just nomenclature, and while it's usually used to refer to time, if you're working with a hypercube you'll probably using it as another spatial dimension. Terminology does this sometimes - for instance, resonance as a chemistry term refers to the delocalization of electrons along a molecule, as a physics term it refers to effects from repeated forces at a particular set of frequencies. Neither of these is the "real resonance".

MLai
2014-04-09, 12:12 AM
It's not like there is a "true 4th dimension". "4th dimension" is just nomenclature, and while it's usually used to refer to time, if you're working with a hypercube you'll probably using it as another spatial dimension.
Okay, put my previous question in a more specific way: When theoretical physicists speak of other/higher dimensions, for example when talking about string theory to the layman... what do they mean by the word "dimension"? Is it the "dimensions" as demonstrated by the hypercubes?
And where does time fit into that definition of dimensions? Ignored, or assigned an arbitrary number in the hierarchy of dimensions?

Jay R
2014-04-09, 12:25 AM
Is there any plausible way for a planet to have Game of Thrones-style seasons? (I.e. summers and winters that last for an irregular number of years).

Not under our physical laws, but with wights, wargs, and the Lord of Light, it's clear that our physical laws don't apply.

So the law that needs to go away is observation of angular momentum. That's what makes gyroscopes work, and that's what keeps the earth's axis tilted consistently (except for very long precession).

So if angular momentum is not preserved consistent, it's possible for the axis to stay turned toward the sun for years, and then turn away from the sun for years. That would only affect one hemisphere, but everything we've seen is northern hemisphere anyway. (the further south you go, the warmer it gets.)

But even that assumes a spherical earth that goes around the sun, and seasons that depend on axial tilt. Suppose the earth is the center of the universe, and the sume goes around it, but not driven by Newton's laws of universal gravitation. It spends several years closer, and then spends several years further away.

So the easy answer is that consistent seasons as we know them depend on our specific physical laws - just like dead people who stay dead and the inability to inhabit the minds of beasts.

Jay R
2014-04-09, 12:33 AM
Okay, put my previous question in a more specific way: When theoretical physicists speak of other/higher dimensions, for example when talking about string theory to the layman... what do they mean by the word "dimension"? Is it the "dimensions" as demonstrated by the hypercubes?

That's a good question. If you come up with an answer, you can have tenure at a major university.

The simple, consistent, and meaningless answer is that a dimension is a measurement that cannot be represented as a combination of the other dimensions.


And where does time fit into that definition of dimensions? Ignored, or assigned an arbitrary number in the hierarchy of dimensions?

In some ways, under Einstein's relativity, the fourth dimension is ct (the speed of light times time). Multiplying by the speed of light makes it a linear dimension. The units are (meters / seconds) * seconds = meters. But that only has meaning for a very small number of equations, and cannot be understood except in equation form.

Do not attempt to understand either relativity or string theory except as a set of equations.

The Extinguisher
2014-04-09, 02:01 AM
Dimension is the worst term in math/physics cause its just so many things.

Mathematical dimensions refer to a number of different things but in general a good overall descriptor is the dimension of a system is the number of independent "things"

Physical dimensions are the same idea but applied to units. Length and time and temperature and charge and things like that.

3D (and the fourth dimension) is about mutual orthogonality and is the number of terms in a vector in space(time)

String theory I believe is the last one, but as far as I can tell it doesn't make any testable predictions so you don't need to worry about it.

Jay R
2014-04-09, 11:44 AM
Dimension is the worst term in math/physics cause its just so many things.

Dimension is a perfectly useful math term. But like all technical jargon, it must be processed in the specific technical sense, not in a generic English sense.

This is true of many technical terms. Variance, integral, term, operation, function, and many other words carry specific meanings in a mathematical context.

But there's nothing special about math here. Any subject develops its own jargon. Home, out, base, safe, foul, and hit have specific meanings in baseball. Class, feat, level, and experience have specific meanings in D&D.

The only problem is trying to understand a complex subject without learning the background. All attempts to understand string theory without a pretty deep grounding in college level math and physics will fail. But don't blame the words.

Grinner
2014-04-09, 03:31 PM
Okay, put my previous question in a more specific way: When theoretical physicists speak of other/higher dimensions, for example when talking about string theory to the layman... what do they mean by the word "dimension"? Is it the "dimensions" as demonstrated by the hypercubes?
And where does time fit into that definition of dimensions? Ignored, or assigned an arbitrary number in the hierarchy of dimensions?

It depends on the subject, I think. Hypercubes, for example, demonstrate spatial dimensions, but if you're talking about time, you're talking about temporal dimensions. I think there's others as well, but I haven't really paid them much attention.


Do not attempt to understand either relativity or string theory except as a set of equations.

Why?

Knaight
2014-04-09, 09:12 PM
Why?

Outside of the friendliest parts of special relativity, they are both hideously complex systems where intuitive understandings aren't going to happen, and will mostly just produce wrong results if applied.

MLai
2014-04-09, 09:27 PM
Outside of the friendliest parts of special relativity, they are both hideously complex systems where intuitive understandings aren't going to happen, and will mostly just produce wrong results if applied.
Actually I don't think that's it at all. Sure, the math is complicated, no doubt. But the underlying concepts can be understood.

What is at work is the opinion that unless you're post-graduate educated in math and physics, it is not possible for you to understand "the reality" behind the math and physics.

Personally I disagree with that, as it is already disproven. Educators routinely leave out the nitty-gritty in order to teach children the basic scientific concepts underlying our reality, concepts that were originally reached via the nitty-gritty.

For example, things like how the concept of the Higgs Boson or gravitational waves in the CBR are explained to the laymen. Sure it may have to be an abridged or approximated version, or hell even the "wrong" version, but helping laymen or children understand such basic concepts only helps the broad endeavor of science.

The key is "Are you an educator?" That is, do you care about enlightening others who do not share your interests and expertise? Do you even know how to do so effectively? Educators carry the double mantle of (1) understanding the field they teach and (2) being interested in shaping and communicating that understanding. We certainly do not need or want all of our scientists to be educators, so it is absolutely no strike against scientists.

Grinner
2014-04-09, 09:35 PM
Outside of the friendliest parts of special relativity, they are both hideously complex systems where intuitive understandings aren't going to happen, and will mostly just produce wrong results if applied.

I'll have to take your word for it.


Actually I don't think that's it at all. Sure, the math is complicated, no doubt. But the underlying concepts can be understood.

What is at work is the opinion that unless you're post-graduate educated in math and physics, it is not possible for you to understand "the reality" behind the math and physics.

Personally I disagree with that, as it is already disproven. Educators routinely leave out the nitty-gritty in order to teach children the basic scientific concepts underlying our reality, concepts that were originally reached via the nitty-gritty.


On the other hand, unless the student is acquainted with the nitty-gritty, he's really just taking the information on faith

See above.

Also, what the hell is a gravitational wave? How can gravity exist as a wave if it's a field predicated on the presence of matter?

MLai
2014-04-10, 12:33 AM
On the other hand, unless the student is acquainted with the nitty-gritty, he's really just taking the information on faith See above.
I'm not asking for anyone to mathematically prove what a dimension is, LOL.

Most of science really is just accepting varying degrees of information on faith. You the researcher is standing atop an enormous pyramid of faith; the only part you absolutely know to be fact are the few bricks you're laying down upon the very top.

Also, what the hell is a gravitational wave? How can gravity exist as a wave if it's a field predicated on the presence of matter?
Because gravity can only travel as fast as the speed of light, so it necessarily must propagate as a wave through spacetime.

Brother Oni
2014-04-10, 02:14 AM
Because gravity can only travel as fast as the speed of light, so it necessarily must propagate as a wave through spacetime.

Isn't that speed restriction only for particles, which light obeys since it's both wave and particle?

I apologise if it'm missing something blatantly obvious as physics isn't my strong point.

factotum
2014-04-10, 03:13 AM
Isn't that speed restriction only for particles, which light obeys since it's both wave and particle?

No, it's for everything. If there were something that didn't obey the speed of light limit then it would be theoretically possible to send information from one place to another at greater than lightspeed, and FTL communications would break relativity in half; it would be possible for an effect to precede its cause in some frames of reference.

(It should be noted that no particle with mass can actually achieve lightspeed, incidentally, because it would require an infinite amount of energy to get it there).

Kato
2014-04-10, 06:40 AM
Without any proper education as an educator, I've come to the realization it can be horrendously difficult to convey concepts which appear simple to oneself to another person. While some people can pretty easily imagine what happens in relativity, special or general, others have a hard time to even get to terms with the fact that you can't move as fast/faster than light. Mostly because it goes totally against your intuition. I think quantum dynamics is even worse in that regard, because imagining particles as wave functions may work well mathematically, but getting any kind of mental image of it is - which is what people do generally - will lead to errors. So you're far better off just taking the equation and math and only apply "reality" when necessary.



Because gravity can only travel as fast as the speed of light, so it necessarily must propagate as a wave through spacetime.
That seems like a pretty poor answer to me, because it hardly explains how being a wave makes you able to move faster. And it raises the question of what kind of wave...
But then I only recall the basic argument that you can get wavelike solutions for Einstein's field equation. (And now I went back to try to read about it in the lecture notes and I remember why I hate general relativity...)

MLai
2014-04-10, 07:07 AM
That seems like a pretty poor answer to me, because it hardly explains how being a wave makes you able to move faster. And it raises the question of what kind of wave...
Actually that's the most "common sensical" way I can think to explain it.
His question was why is it a wave if it's a field. He was making the common mistake of thinking that gravity is instantaneous.
I was saying that gravity is not instantaneous; it travels at the speed of light.
Brian Greene gave a good scenario for easy visualization:
Imagine the sun magically blinked out of existence right this very instant. It will take 8 mins 20 secs for the Earth to suddenly feel of loss of the Sun's attraction. At the exact same time the sun suddenly goes dark to us, Earth flies off into space.

So since gravity isn't instantaneous, what does the "front" of a gravity propagation "look like"? Well, it's going to take the "shape" of an expanding spherical wavefront. What else can it look like?

Kato
2014-04-10, 12:15 PM
The matter isn't with the propagation speed, (obviously) this has to be finite in accordance with GR, but why a wave? A wave is more than just "something that comes towards you", isn't it? It's hard enough to get a "macroscopic" explanation of why the energy of a photon is tied to its frequency, what would the frequency of a gravitational wave even represent?
I guess I might as well follow up with a question since thinking about this made me question it: Obviously, all other force fields need to propagate with c as well. So, is there an analogue for gravitational waves for strong and weak force fields? I guess for charges there is so there must be..

Yora
2014-04-11, 01:59 PM
Okay, here is a weird one:

Say you have a couple of brushes that you are going to clean in a jar with water.

You could either put them all into the jar at once, or only clean one brush at a time.

But when you clean a brush, the water gets dirty and isn't able to clean brushes as well as fresh water.


Now the question is this: Does it make an overall difference to clean all the brushes at once or one at a time?


The first brush you put into the jar will be the most clean, because there barely is any paint in the water now. It will be cleaner than if you had cleaned all the brushes at once.
But when the last brush is cleaned individually, there is there the same amount of paint in the water as if you cleaned all at once, or is the water dirtier than it would have been?

Does it make a difference in regard to how much paint will be left in all the brushes combined?

Tirunedeth
2014-04-11, 03:10 PM
Okay, here is a weird one:

Say you have a couple of brushes that you are going to clean in a jar with water.

You could either put them all into the jar at once, or only clean one brush at a time.

But when you clean a brush, the water gets dirty and isn't able to clean brushes as well as fresh water.


Now the question is this: Does it make an overall difference to clean all the brushes at once or one at a time?


The first brush you put into the jar will be the most clean, because there barely is any paint in the water now. It will be cleaner than if you had cleaned all the brushes at once.
But when the last brush is cleaned individually, there is there the same amount of paint in the water as if you cleaned all at once, or is the water dirtier than it would have been?

Does it make a difference in regard to how much paint will be left in all the brushes combined?

Well, here's a shot at this. Let N be the number of brushes, P the amount of paint initially on each brush, and C be a constant describing how much paint is left on a brush after cleaning it (i.e., if there is X paint in the water, the amount of paint left on the brush after cleaning it is C*X). We'll also assume that the amount of paint left on the brush is small relative to the amount of paint left in the water.

For sequential cleaning, the k-th brush has k*P paint in the water; therefore, the amount of paint left on it afterwards is k*P*C. Summing over k from 1 to N gives us 1*P*C + 2*P*C + ... + N*P*C = (1 + 2 + ... + N)*P*C. The first and last terms of the series 1 + 2 + ... + N sum to N + 1, as do the second and second-to-last, and so forth. There are N/2 of these combinations, so the total amount of paint on the brushes after cleaning in the sequential case is 1/2*N*(N+1)*P*C.

For batch cleaning, things are much simpler. There is N*P paint in the water, so each brush takes N*P*C paint with it. There are N brushes, so the total amount of paint on all brushes is N2*P*C.

For large N, 1/2*N*(N+1) is less than N2, since N+1 ~= N, which gives us 1/2*N*(N+1) ~= 1/2*N2. Looking at a graph of the two functions, it seems that N2 is true for all integers except 0 and 1, where they are equal.

This kind of makes intuitive sense. The final brush in the sequential case will be the dirtiest brush, and should be equally dirty to each brush in the batch case. Since each brush in the sequential case is less dirty than any of the brushes in the batch case, this means that the amount of paint on all the brushes in the sequential case must be less than that in the batch case.

I was a little concerned at first that there is an N2 dependence for the paint on the brushes, while the paint in the water goes as N, but I think that this is just because of the simplifying assumptions I made. I'm not quite sure how they break down, but I think either there will be so much paint in the water at some point that the brushes aren't cleaned, or the brushes reach a point where they don't actually pick up any more paint when the amount of paint in the water is increased.

Max™
2014-04-11, 04:30 PM
The matter isn't with the propagation speed, (obviously) this has to be finite in accordance with GR, but why a wave? A wave is more than just "something that comes towards you", isn't it? It's hard enough to get a "macroscopic" explanation of why the energy of a photon is tied to its frequency, what would the frequency of a gravitational wave even represent?
I guess I might as well follow up with a question since thinking about this made me question it: Obviously, all other force fields need to propagate with c as well. So, is there an analogue for gravitational waves for strong and weak force fields? I guess for charges there is so there must be..

Massless force carriers propagate at c, in fact something which is massless must propagate at c.

Gluons are confined, so the propagation velocities are... complicated, and W/Z bosons are massive so they are limited to sub-luminal velocities.

Gravity waves are best understood as a result of very large and very massive bodies undergoing very dramatic motions.

Black holes colliding have... interesting effects on spacetime in their vicinity, and the rapidly decreasing distances between them as they orbit need some way to carry the momentum away. Asking the question "what could escape from a pair of colliding black holes?" gives the answer "gravitational waves".

Ah, these might help, the animations on this page: http://en.wikipedia.org/wiki/Gravitational_wave#Effects_of_a_passing_gravitatio nal_wave as the caption says, represent a ring of free floating particles being distorted by a passing gravitational wave.

QNLA
2014-04-11, 08:30 PM
String theory is basically a very general theory, and is an extension of quantum field theory where instead of looking at point particles we are looking at a string. The mass of the vibrating string depends on the vibrations. And quantum mechanics means that there is a minimum amount of vibrations allowed for each dimension the string can vibrate in. However because of uncertainty there is a component which reduces the mass (really mass squared). However if that component is larger then the ground state vibrations then we get imaginary mass and faster then light particles in our theory which doesn't make sense so to counteract that string theory requires a minimum of 25 spatial dimensions to make the theory sensible. If we add supersymmetry we only need 9 spatial dimensions for the theory to be sensible.

The compactified dimensions in string theory are easier to make sense of if you think about the distance from any one point on a cylinder to another. If the two points are quite different along the height, then the positions of the other dimension (around the cylinder) cannot effect the distance significantly regardless of positions. So we can describe these positions by only the height and we've hidden the second dimension.

As for the energy of a photon, in special relativity the spacetime distance between two points is given by:
(ds)^2 = -(dt)^2 + (dx)^2+(dy)^2+(dz)^2

think of the d as a delta (change in) and I've also set the speed of light to 1.

Light is massless so it travels along a null geodesic, so ds=0. Our momentum is like our small changes in space for a step and our energy is like a small change in time, for a step. So because we are going along a null geodesic we have that the energy of a massless particle must be proptional to the magnitude of its momentum.


hopefully that makes sense (did this on a train). As for questions, there seems to be a number of mathematicians here. Could anyone discribe to me what the meaning of a function/field/ect being memoryless means mathematically. I've heard about it in both maths and physics but only have a rough physical idea of what it means.

Max™
2014-04-12, 04:00 AM
Uh, I think there are a few meanings, one is related to Markov as I recall, where the present state of a system is all that is required to make predictions about future states, hence it is memoryless because the past states don't change the outcome.

the_druid_droid
2014-04-14, 01:56 AM
Also, what the hell is a gravitational wave? How can gravity exist as a wave if it's a field predicated on the presence of matter?

Same way electromagnetic waves exist; there's an EM field and it's predicated on charges in just the way you've described for gravity. In fact, I believe the similarity of the basic idea led Einstein to try for a geometrical theory of electromagnetism similar to SR/GR for a while, though that could be apocryphal, and didn't pan out in any case.


Isn't that speed restriction only for particles, which light obeys since it's both wave and particle?

I apologise if it'm missing something blatantly obvious as physics isn't my strong point.

It's basically a restriction on information. So massless or massive particles are all constrained, although talking about how those restrictions come into play with things like entanglement in quantum mechanics gets somewhat confusing.


Uh, I think there are a few meanings, one is related to Markov as I recall, where the present state of a system is all that is required to make predictions about future states, hence it is memoryless because the past states don't change the outcome.

Yep, that's the one I'm familiar with as well. Although I think the specific criteria for determining if Markovian is a decent approximation vary a bit from application to application.

noparlpf
2014-04-14, 10:17 AM
RE: Irregular seasons
More importantly, how would plants and animals evolve in a world with irregular seasons? Would estrus follow the same cycles? Would deciduous trees develop in the same way? How many growing seasons would there be in a summer?

factotum
2014-04-14, 02:00 PM
RE: Irregular seasons
More importantly, how would plants and animals evolve in a world with irregular seasons? Would estrus follow the same cycles? Would deciduous trees develop in the same way? How many growing seasons would there be in a summer?

Well, no, realistically. All creatures would probably have to be like humans and not have a fixed breeding season, and plants would probably develop in a similar way to those Earthly ones that stay dormant in deserts for years and then bloom, spore and die all in the course of the few hours when it actually rains. In fact, figuring out how plants work would be the key to figuring out the rest, since plant life forms the basis for 99% of life on Earth and we have no reason to suppose it'll be much different elsewhere.

(I recall watching a TV program years ago about what life would be like on a planet orbiting a flare star--everything had to develop to be able to survive the bursts of radiation when the star flared. There were massive forests, but the trees were all a type of animal rather than plant life as we know it. Wish I could remember what the planet was called, because it was supposedly thought up by some of the foremost authorities in possible extra-terrestrial life).

noparlpf
2014-04-14, 02:46 PM
But plants aren't the basis for animal life. The two developed separately from different ancestral eukaryotic lines. Multicellularity probably developed independently too, but I'm not sure of that.

factotum
2014-04-15, 01:31 AM
But plants aren't the basis for animal life. The two developed separately from different ancestral eukaryotic lines.

I didn't mean they developed from the same ancestor, I mean they're the basis in terms of the food chain. Even the biggest, most vicious predator is ultimately eating animals that subsist on plants, at least on land (in the sea it would be algae).

Eldan
2014-04-15, 01:38 AM
But plants aren't the basis for animal life. The two developed separately from different ancestral eukaryotic lines. Multicellularity probably developed independently too, but I'm not sure of that.

People assume that evolution is a strict progression of primitive to modern characteristics, but actually — from an indepth, modern viewpoint — it's more like a big ball of wibbly-wobbly... .evolutiony-vutiony.. stuff.

Or rather, things get messy, blurry and chaotic around the major early branching points. There's the theory that everything was just swapping genes with everything else back then. Multicellularity may have happened several times (algal colonies and sponges, probably) or it may not.

Kato
2014-04-16, 03:36 AM
RE: Irregular seasons
More importantly, how would plants and animals evolve in a world with irregular seasons? Would estrus follow the same cycles? Would deciduous trees develop in the same way? How many growing seasons would there be in a summer?
Kind of related question for the more biology savvy: How much are plants relying on internal clocks compared to just reacting to the environment? e.g. if I took a plant that's been growing on the northern hemisphere and brought it to the southern, would it get screwed up or would it not care? Or say, if I put it in a totally dark room and instead of giving it normal sunlight would blast it with artificial sunlight all night. I'd guess the matter is much more one of the external input than any kind of internal one, so Westerosi plants would just need to be able to live through longer winters waiting for warmth... animals would either migrate stronger and/or more of the would be resting or in stasis, I guess.


People assume that evolution is a strict progression of primitive to modern characteristics, but actually — from an indepth, modern viewpoint — it's more like a big ball of wibbly-wobbly... .evolutiony-vutiony.. stuff.

Hah, here have an internet cookie :smallbiggrin:
I always wonder if since evolution by trial and error works so well we could just try to get artificial intelligence by making a program which edits its own code and let it run sufficiently long. Well, it would probably take really long.

SiuiS
2014-04-16, 04:23 AM
So, everything discovered before QM cannot possibly have quantum effects. You're being arbitrary. You're dividing physics into two categories based on historical accident.

And that's my point. Everything in our universe is based on QM and Relativity, not classical physics. Classical physics is an approximation and every time you use it, you must ask if it's applicable. Or does it give the wrong answer because it is the simplified version.

I'm crazy late, but I believe this is an example of a heuristic; a pattern observed that holds true, broadly, Ben if it isn't 100% accurate to the equations involved.

Also, what font is in your signature? I would love a way to pepper some RP posts with upside down letters!

Kato: I can't do justice to the specifics but a friend of mine has been pondering stuff like that. He believes the reason we don't have AI is because it cannot emerge directly from binary code. Some fundamental factor is missing. You would have to use binary code to make a software model of a nonbinary code, and that could create artificial intelligence (in theory; in spitballing and don't fully grok the mechanics enough to be sure), but simple yes/no programming like we have will not ever get to intelligence.

Kato
2014-04-16, 04:44 AM
Kato: I can't do justice to the specifics but a friend of mine has been pondering stuff like that. He believes the reason we don't have AI is because it cannot emerge directly from binary code. Some fundamental factor is missing. You would have to use binary code to make a software model of a nonbinary code, and that could create artificial intelligence (in theory; in spitballing and don't fully grok the mechanics enough to be sure), but simple yes/no programming like we have will not ever get to intelligence.

It's not really my area of expertise but while I'll admit it might be much easier to create an AI by other means I don't think it's impossible to make one with binaries. From a very pragmatic point of view, all our neurons do is fire or not fire (and be connected or not be connected, and a bunch of other stuff, but it all is or at least seems pretty binary, i guess.) Then again, modelling a brain 1:1 as a network model is likely not the most efficient way of making an AI.

Fortuna
2014-04-16, 05:30 AM
Kato: I can't do justice to the specifics but a friend of mine has been pondering stuff like that. He believes the reason we don't have AI is because it cannot emerge directly from binary code. Some fundamental factor is missing. You would have to use binary code to make a software model of a nonbinary code, and that could create artificial intelligence (in theory; in spitballing and don't fully grok the mechanics enough to be sure), but simple yes/no programming like we have will not ever get to intelligence.

(Hiya, Starry. Long time no see. :smallsmile:)

Depending on how you interpret it, that looks either like a rather trivial statement or one that runs afoul of Turing. I'd be really interested to hear it unpacked - perhaps you could ask your friend for more information?

TuggyNE
2014-04-16, 06:07 AM
I always wonder if since evolution by trial and error works so well we could just try to get artificial intelligence by making a program which edits its own code and let it run sufficiently long. Well, it would probably take really long.

I've done some work on that, actually. It tends, in the medium term, toward silly trivial optimizations like disabling checks on reproduction, but without actually adding much that's interesting.

(I used a set of JS scripts run locally, each copying itself to a pair of descendents with modifications to the source and then being deleted by a master script; any script that failed to run due to syntactic errors was still deleted, of course.)

What I would be interested in seeing, though, is a formal analysis of generations taken, in Big-O notation, for changes of a given magnitude in genomes of different lengths. Offhand it seems as though longer genomes would cause problems from conflation, since the entire genome is tested fuzzily for fitness at the same time, so multiple mutations (some positive, some negative) would tend to cancel out their appropriate survival changes whenever present together, and therefore natural selection would become superlinearly fuzzier. Of course, running Big-O(n log n) algorithms or whatever with n > 1e+9 and C >> 10 years is no joke, even parallelizing with millions or billions of organisms, so maybe there's something I'm missing.

factotum
2014-04-16, 06:16 AM
I always wonder if since evolution by trial and error works so well we could just try to get artificial intelligence by making a program which edits its own code and let it run sufficiently long. Well, it would probably take really long.

I'm pretty sure it's been tried. I suspect part of the problem, though, is that you would need some way of determining that the random change you just made is actually better than the previous interation--in real life this is handled quite neatly because the "better" traits naturally tend to survive better, but how would you handle that in a virtual environment?

Grinner
2014-04-16, 07:33 AM
Hah, here have an internet cookie :smallbiggrin:
I always wonder if since evolution by trial and error works so well we could just try to get artificial intelligence by making a program which edits its own code and let it run sufficiently long. Well, it would probably take really long.

Well...That is the basic principle behind evolutionary algorithms...The most fundamental problem is that evolutionary algorithms are designed to solve single problems, whereas sapient lifeforms do far more than problem solving. By incorporating neural networks into the mix, it is possible to create a decision-making capability, but there's still an essential element missing. I guess you could call it creativity, but impetus might also be a valid term.

Edit: Evolution is more than just random changes. It's adaptation, a response to environmental pressures. Problematically, most computer programs exist in a vacuum, so they lack an environment. Humans, on the other hand, learn by interacting with, or at the very least observing, their environment.

In this sense, Dwarf Fortress is probably the closest thing we've got. If somebody were to replace the AI code with the methods I mentioned, restructure it to accommodate multithreading, and run the program on one of those eight-core processors (or perhaps a mountain of Raspberry Pi's), it might get close.

noparlpf
2014-04-16, 08:18 AM
Do we really want a sentient version of Dwarf Fortress?

Frozen_Feet
2014-04-16, 10:19 AM
Evolution is the non-random survival of random variables. The problem, like correctly stated above, is the simplicity of environment: the "environment" of most evolutionary algorithms consists of a single (or very few) problems, so they peak very quickly. Even worse, evolutionary algorithms can get stuck in local peaks. I lack the expertise to properly describe the process, but basically, in addition to the "best" solution, there also several "very good" solutions from which it is impossible to get to the "best" one. To get to the best solution, one would hence need to abandon all progress made towards the very good solution and start again from scratch.

In order to prevent such "evolutionary deadends", our "environment" or set of problems would have to be dynamic and potentially include a random factor that introduces new problems to optimize towards so the process won't halt. Taking inspiration from nature, we would probably want to pit several algorithms against each other while they are also pitted against a set of static problems. If I recall right, 80% of context from evolution of real organisms comes from other organisms. Inorganic environment accounts only for 20% of selective pressures.

Sith_Happens
2014-04-16, 11:30 AM
So I just read through all of this thread, and in the hypothetical eventually that I actually get around to expanding my signature in some way (either via link or spoiler box) so as to include more quotes, about half of the quotes will be coming from this thread.:smallbiggrin:

Sallera
2014-04-16, 04:37 PM
Also, what font is in your signature? I would love a way to pepper some RP posts with upside down letters!

Upside down letters are Unicode ʎɹəʞɔ׀ɹʇ rather than a specific font - if you look at the character map (Accessories->System Tools->Character Map on a Windows system) you can find the glyphs for a lot of upside down letters (or reasonable approximations thereof). I'm not sure the set is complete in most fonts, though.

Knaight
2014-04-16, 04:49 PM
In order to prevent such "evolutionary deadends", our "environment" or set of problems would have to be dynamic and potentially include a random factor that introduces new problems to optimize towards so the process won't halt. Taking inspiration from nature, we would probably want to pit several algorithms against each other while they are also pitted against a set of static problems. If I recall right, 80% of context from evolution of real organisms comes from other organisms. Inorganic environment accounts only for 20% of selective pressures.

It's worth noting that genetic algorithms have still produced some pretty impressive results, despite generally lacking this sort of thing. The recent NASA antenna comes to mind here - it's highly effective, despite being pretty weird.

Grinner
2014-04-16, 08:02 PM
Evolution is the non-random survival of random variables. The problem, like correctly stated above, is the simplicity of environment: the "environment" of most evolutionary algorithms consists of a single (or very few) problems, so they peak very quickly. Even worse, evolutionary algorithms can get stuck in local peaks. I lack the expertise to properly describe the process, but basically, in addition to the "best" solution, there also several "very good" solutions from which it is impossible to get to the "best" one. To get to the best solution, one would hence need to abandon all progress made towards the very good solution and start again from scratch.

In order to prevent such "evolutionary deadends", our "environment" or set of problems would have to be dynamic and potentially include a random factor that introduces new problems to optimize towards so the process won't halt. Taking inspiration from nature, we would probably want to pit several algorithms against each other while they are also pitted against a set of static problems. If I recall right, 80% of context from evolution of real organisms comes from other organisms. Inorganic environment accounts only for 20% of selective pressures.

Are we talking about AI or algorithms now?

The two are quite different.

Frozen_Feet
2014-04-17, 12:58 AM
We're talking of making an AI through using multiple evolutionary algorithms. So both.

Radar
2014-04-17, 02:04 AM
If we are talking about AI, then it would be good to be specific: are we talking about specialised and self-learning semi-autonomous system, or sentience? The first one is already there and it does rely on genetic algorithms. The second one IMO is not reachable in classical computers for a very specific reason: sentience is non-algorithmic. It was well explained by Roger Penrose in The Emperor's New Mind (great book by the way) with examples such as Gödel's incompleteness theorems - there is no proof for it (which is the point of the theorem), but we can intuitively understand, that it is indeed true. Therefore, we can think in non-algorithmic ways.

That's beside the main problem: we have no means of objectively checking sentience. For an outside observer there is no difference between a sentient being and a non-sentient one as long as they both appear to be sentient (see the idea behind Turing test and it's weakness). For the most part, we can only be sure of our own consciousness and simply assume that others share that quality.

Eldan
2014-04-17, 03:02 AM
I fail to see what the difference would be between a "truly sapient/sentient" system and one that's just sufficiently good at "mimicking" it.

I'd say by the point where computers can fool us into thinking they are sapient, they might as well be. Sapience is not some kind of mystical quality that is bestowed upon certain systems.